ABDOMEN EMRCS Flashcards

1
Q

A 53 year old man undergoes a reversal of a loop colostomy. He recovers well and is discharged home. He is readmitted 10 days later with symptoms of vomiting and colicky abdominal pain. On examination he has a swelling of the loop colostomy site and it is tender. What is the most likely underlying diagnosis?
A-Haematoma
B-Intra abdominal adhesions
C-Anastomotic leak
D-Anastomotic stricture
E-Obstructed incisional hernia

A

Theme from September 2011 Exam
In this scenario the most likely diagnosis would be obstructed incisional hernia. The tender swelling coupled with symptoms of obstruction point to this diagnosis. Prompt surgical exploration is warranted. Loop colostomy reversals are at high risk of this complication as the operative site is at increased risk of the development of post operative wound infections.
Acute incisional hernia
Any surgical procedure involving entry into a cavity containing viscera may be complicated by post operative hernia
The abdomen is the commonest site
The deep layer of the wound has usually broken down, allowing internal viscera to protrude through
Management is dictated by the patients clinical status and the timing of the hernia in relation to recent surgery
Bowel obstruction or tenderness at the hernia site both mandate early surgical intervention to reduce the risk of bowel necrosis Mature incisional hernias with a wide neck, and no symptoms, may be either left or listed for elective repair
Risk factors for the development of post operative incisional hernias include; post operative wound infections, long term steroid use, obesity and chronic cough

How well did you know this?
1
Not at all
2
3
4
5
Perfectly
2
Q

Theme: Abdominal stomas
A. End ileostomy
B. End colostomy
C. Loop ileostomy
D. Loop colostomy
E. End jejunostomy
F. Loop jejunostomy
G. Caecostomy
For each of the following scenarios, please select the most appropriate type of stoma to be constructed. Each option may be selected once, more than once or not at all.

A 56 year old man is undergoing a low anterior resection for carcinoma of the rectum. A primary anastomosis is planned.

A

Loop ileostomy
Theme from April 2014 Exam
Colonic resections with an anastomosis below the peritoneal reflection may have an anastomotic leak rate (both clinical and radiological) of up to 15%. Therefore most surgeons will defunction such an anastomosis to reduce the clinical severity of an anastomotic leak. A loop ileostomy will achieve this end point and is relatively easy to reverse.

How well did you know this?
1
Not at all
2
3
4
5
Perfectly
3
Q

Theme: Abdominal stomas
A. End ileostomy
B. End colostomy
C. Loop ileostomy
D. Loop colostomy E. End jejunostomy F. Loop jejunostomy G. Caecostomy
For each of the following scenarios, please select the most appropriate type of stoma to be constructed. Each option may be selected once, more than once or not at all.

A 23 year old man with uncontrolled ulcerative colitis is undergoing an emergency sub total colectomy.

A

The correct answer is End ileostomy
Following a sub total colectomy the immediate surgical options include an end ileostomy or ileorectal anastomosis. In the emergency setting an ileorectal anastomosis would be unsafe.

How well did you know this?
1
Not at all
2
3
4
5
Perfectly
4
Q

Theme: Abdominal stomas
A. End ileostomy
B. End colostomy
C. Loop ileostomy
D. Loop colostomy E. End jejunostomy F. Loop jejunostomy G. Caecostomy
For each of the following scenarios, please select the most appropriate type of stoma to be constructed. Each option may be selected once, more than once or not at all.

A 63 year old women presents with large bowel obstruction. On examination she has a carcinoma 10cm from the anal verge.

A

The correct answer is Loop colostomy
Large bowel obstruction resulting from carcinoma should be resected, stented or defunctioned. The first two options typically apply to tumours above the peritoneal reflection. Lower tumours should be defunctioned with a loop colostomy and then formal staging undertaken prior to definitive surgery. An emergency attempted rectal resection carries a high risk of involvement of the circumferential resection margin and is not recommended.

How well did you know this?
1
Not at all
2
3
4
5
Perfectly
5
Q

Theme: Appendicitis
A. Colonoscopy
B. MRIAbdomen
C. Appendicectomy
D. AbdominalCTscan
E. Barium enema
F. Exploratory laparotomy
G. Conservative management with intravenous antibiotics H. Re-assure and discharge
I. Abdominal ultrasound scan
Please select the most appropriate management option for the following patients. Each option may be used once, more than once or not at all.

A 24 year old man presents with a 10 day history of right sided abdominal pain. Prior to this he was well. On examination he has a low grade fever and a mass palpable in the right iliac fossa. The rest of his abdomen is soft. An abdominal USS demonstrates matted bowel loops surrounding a thickened appendix.

A

The correct answer is Conservative management with intravenous antibiotics
This man is likely to have an appendix mass. There is no history suggestive of inflammatory bowel disease. These are usually managed without surgery, especially in the absence of peritoneal signs. Broad spectrum antibiotics are required. In the past an interval appendicectomy was performed. This is rare now and in most cases the process resolves with fibrosis of the appendix.

How well did you know this?
1
Not at all
2
3
4
5
Perfectly
6
Q

Theme: Appendicitis
A. Colonoscopy
B. MRIAbdomen
C. Appendicectomy
D. AbdominalCTscan
E. Barium enema
F. Exploratory laparotomy
G. Conservative management with intravenous antibiotics H. Re-assure and discharge
I. Abdominalultrasoundscan
Please select the most appropriate management option for the following patients. Each option may be used once, more than once or not at all.

A 22 year old man presents with a 48 hour history of right iliac fossa pain. On examination he has a low grade pyrexia and is tender with voluntary guarding in the right iliac fossa. His blood tests reveal a WCC of 13 and a CRP of 6. A urine dipstick is positive for leucocytes.

A

Appendicectomy
This is a typical history for acute appendicitis and in a young male, few differentials would be compatible with this history and signs. Whilst inflammatory markers may be raised this is by no means universal. Further imaging will delay treatment and is unlikely to alter the eventual surgical outcome.

How well did you know this?
1
Not at all
2
3
4
5
Perfectly
7
Q

Theme: Appendicitis
A. Colonoscopy
B. MRIAbdomen
C. Appendicectomy
D. AbdominalCTscan
E. Barium enema
F. Exploratory laparotomy
G. Conservative management with intravenous antibiotics H. Re-assure and discharge
I. Abdominalultrasoundscan
Please select the most appropriate management option for the following patients. Each option may be used once, more than once or not at all.

A 63 year old man presents with a 48 hour history of right iliac fossa pain. On examination he has a low grade pyrexia and is tender with some voluntary guarding in the right iliac fossa. Some of his blood tests are reproduced below:
Hb 8.1 WCC 13.8 Platelets 438 Albumin 22 CRP 24

A

The correct answer is Abdominal CT scan
This man’s investigations point to a more longstanding disease process (Hb and albumin), right sided colonic cancer being the most likely. For this reason a CT scan is a sensible option as it will adjust the surgical planning.

How well did you know this?
1
Not at all
2
3
4
5
Perfectly
8
Q

Theme: Acute abdominal pain
A. Ruptured abdominal aortic aneurysm B. Perforated peptic ulcer
C. Perforated appendicitis
D. Mesenteric infarction
E. Small bowel obstruction
F. Large bowel obstruction G. Pelvic inflammatory disease
H. Mesenteric adenitis I. Pancreatitis
J. Noneoftheabove
Please select the most likely cause of abdominal pain for the scenario given. Each option may be used once, more than once or not at all.

A 75 year old man is admitted with sudden onset severe generalised abdominal pain, vomiting and a single episode of bloody diarrhoea. On examination he looks unwell and is in uncontrolled atrial fibrillation. Although diffusely tender his abdomen is soft.

A

The correct answer is Mesenteric infarction
In mesenteric infarction there is sudden onset of pain together with vomiting and occasionally passage of bloody diarrhoea. The pain present is usually out of proportion to the physical signs.

How well did you know this?
1
Not at all
2
3
4
5
Perfectly
9
Q

Theme: Acute abdominal pain
A. Ruptured abdominal aortic aneurysm B. Perforated peptic ulcer
C. Perforated appendicitis
D. Mesenteric infarction
E. Smallbowelobstruction
F. Largebowelobstruction G. Pelvic inflammatory disease
H. Mesenteric adenitis I. Pancreatitis
J. Noneoftheabove
Please select the most likely cause of abdominal pain for the scenario given. Each option may be used once, more than once or not at all.

A 19 year old lady is admitted with lower abdominal pain. On examination she is diffusely tender. A laparoscopy is performed and at operation multiple fine adhesions are noted between the liver and abdominal wall. Her appendix is normal.

A

The correct answer is Pelvic inflammatory disease
This is Fitz Hugh Curtis syndrome in which pelvic inflammatory disease (usually Chlamydia) causes the formation of fine peri hepatic adhesions.

How well did you know this?
1
Not at all
2
3
4
5
Perfectly
10
Q

Theme: Acute abdominal pain
A. Ruptured abdominal aortic aneurysm B. Perforated peptic ulcer
C. Perforated appendicitis
D. Mesenteric infarction
E. Smallbowelobstruction
F. Largebowelobstruction G. Pelvic inflammatory disease
H. Mesenteric adenitis I. Pancreatitis
J. Noneoftheabove
Please select the most likely cause of abdominal pain for the scenario given. Each option may be used once, more than once or not at all.

A 78 year old man is walking to the bus stop when he suddenly develops severe back pain and collapses. On examination he has a blood pressure of 90/40 and pulse rate of 110. His abdomen is distended and he is obese. Though tender his abdomen itself is soft.

A

The correct answer is Ruptured abdominal aortic aneurysm
This will be a retroperitoneal rupture (anterior ones generally don’t survive to hospital). The debate regarding CT varies, it is the authors opinion that a systolic BP of <100mmHg at presentation mandates immediate laparotomy.

How well did you know this?
1
Not at all
2
3
4
5
Perfectly
11
Q

Theme: Management of splenic trauma
A. Splenectomy
B. Angiography
C. CT Scan
D. Admit for bed rest and observation E. Ultrasound scan
F. Splenic conservation G. MRIoftheabdomen
Please select the most appropriate intervention for the scenario given. Each option may be used once, more than once or not at all.

A 7 year old boy falls off a wall the distance is 7 feet. He lands on his left side and there is left flank bruising. There is no haematuria. He is otherwise stable and haemoglobin is within normal limits.

A

The correct answer is Ultrasound scan
This will demonstrate any overt splenic injury. A CT scan carries a significant dose of radiation. In the absence of haemodynamic instability or other major associated injuries the use of USS to exclude intraabdominal free fluid (blood) would seem safe when coupled with active observation. An USS will also show splenic haematomas.

How well did you know this?
1
Not at all
2
3
4
5
Perfectly
12
Q

Theme: Management of splenic trauma
A. Splenectomy
B. Angiography
C. CT Scan
D. Admit for bed rest and observation E. Ultrasound scan
F. Splenic conservation G. MRIoftheabdomen
Please select the most appropriate intervention for the scenario given. Each option may be used once, more than once or not at all.

A 42 year old motorcyclist is involved in a road traffic accident. A FAST scan in the emergency department shows free intrabdominal fluid and a laparotomy is performed. At operation there is evidence of small liver laceration that has stopped bleeding and a tear to the inferior pole of the spleen.

A

Splenic conservation
As minimum damage, attempt conservation.

How well did you know this?
1
Not at all
2
3
4
5
Perfectly
13
Q

Theme: Management of splenic trauma
A. Splenectomy
B. Angiography
C. CT Scan
D. Admit for bed rest and observation E. Ultrasound scan
F. Splenic conservation G. MRIoftheabdomen
Please select the most appropriate intervention for the scenario given. Each option may be used once, more than once or not at all.

An 18 year old man is involved in a road traffic accident. A CT scan shows disruption of the splenic hilum and a moderate sized perisplenic haematoma.

A

The correct answer is Splenectomy
Hilar injuries usually mandate splenectomy. The main risk with conservative management here is that he will rebleed and with hilar injuries this can be dramatic.

How well did you know this?
1
Not at all
2
3
4
5
Perfectly
14
Q

Theme: Gastrointestinal bleeding
A. Haemorrhoids
B. Meckels diverticulum C. Angiodysplasia
D. Colonic cancer
E. Diverticular bleed
F. Ulcerativecolitis
G. Ischaemic colitis
Please select the most likely cause of colonic bleeding for the scenario given. Each option may be used once, more than once or not at all

A 73 year old lady is admitted with a brisk rectal bleed. She is otherwise well and the bleed settles. On examination her abdomen is soft and non tender. Elective colonoscopy shows a small erythematous lesion in the right colon, but no other abnormality.

A

A n g io d y s p la s ia
Angiodysplasia can be difficult to identify and treat. The colonoscopic stigmata are easily missed by poor bowel preparation.

How well did you know this?
1
Not at all
2
3
4
5
Perfectly
15
Q

Theme: Gastrointestinal bleeding
A. Haemorrhoids
B. Meckels diverticulum C. Angiodysplasia
D. Colonic cancer
E. Diverticular bleed
F. Ulcerativecolitis
G. Ischaemic colitis
Please select the most likely cause of colonic bleeding for the scenario given. Each option may be used once, more than once or not at all

A 23 year old man complains of passing bright red blood rectally. It has been occurring over the past week and tends to occur post defecation. He also suffers from pruritus ani.

A

Haemorrhoids
Classical haemorrhoidal symptoms include bright red rectal bleeding, it typically occurs post defecation and is noticed on the toilet paper and in the toilet pan. It is usually painless, however, thrombosed external haemorrhoids may be very painful.

How well did you know this?
1
Not at all
2
3
4
5
Perfectly
16
Q

Theme: Gastrointestinal bleeding
A. Haemorrhoids
B. Meckels diverticulum C. Angiodysplasia
D. Colonic cancer
E. Diverticular bleed
F. Ulcerativecolitis
G. Ischaemic colitis
Please select the most likely cause of colonic bleeding for the scenario given. Each option may be used once, more than once or not at all

A 63 year old man presents with episodic rectal bleeding the blood tends to be dark in colour and may be mixed with stool. His bowel habit has been erratic since an abdominal aortic aneurysm repair 6 weeks previously.

A

The correct answer is Ischaemic colitis
The inferior mesenteric artery may have been ligated and being an arteriopath collateral flow through the marginal may be imperfect.

How well did you know this?
1
Not at all
2
3
4
5
Perfectly
17
Q

Theme: Surgical signs
A. Rovsing’s sign
B. Boas’ sign
C. Psoas stretch sign D. Cullen’s sign
E. Grey-Turner’ssign F. Murphy’ssign
G. None of the above
Please select the most appropriate eponymous abdominal sign for the scenario given. Each option may be used once, more than once or not at all.

Severe acute peri-umbilical bruising in the setting of acute pancreatitis.

A

The correct answer is Cullen’s sign
Cullens sign occurs when there has been intraabdominal haemorrage. It is seen in cases of severe haemorrhagic pancreatitis and is associated with a poor prognosis. It is also seen in other cases of intraabdominal haemorrhage (such as ruptured ectopic pregnancy).

How well did you know this?
1
Not at all
2
3
4
5
Perfectly
18
Q

Theme: Surgical signs
A. Rovsing’s sign
B. Boas’ sign
C. Psoas stretch sign D. Cullen’s sign
E. Grey-Turner’ssign F. Murphy’ssign
G. None of the above
Please select the most appropriate eponymous abdominal sign for the scenario given. Each option may be used once, more than once or not at all.

In acute cholecystitis there is hyperaesthesia beneath the right scapula.

A

The correct answer is Boas’ sign
Boas sign refers to this hyperaesthesia. It occurs because the abdominal wall innervation of this region is from the spinal roots that lie at this level.

How well did you know this?
1
Not at all
2
3
4
5
Perfectly
19
Q

Theme: Surgical signs
A. Rovsing’s sign
B. Boas’ sign
C. Psoas stretch sign D. Cullen’s sign
E. Grey-Turner’ssign F. Murphy’ssign
G. None of the above
Please select the most appropriate eponymous abdominal sign for the scenario given. Each option may be used once, more than once or not at all.

In appendicitis palpation of the left iliac fossa causes pain in the right iliac fossa.

A

The correct answer is Rovsing’s sign
Rovsings sign elicits tenderness because the deep palpation induces shift of the appendix (which is inflamed) against the peritoneal surface. This has somatic innervation and will therefore localise the pain. It is less reliable in pelvic appendicitis and when the appendix is truly retrocaecal

How well did you know this?
1
Not at all
2
3
4
5
Perfectly
20
Q

Theme: Surgical access
A. Gridiron
B. Lanz
C. McEvedy
D. Midline abdominal E. Rutherford Morrison F. Battle(abdominal) G. Lower midline
Please select the most appropriate incision for the procedure required. Each option may be used once, more than once or not at all.

A 78 year old lady is admitted with a tender lump in her right groin. It is within the femoral triangle and there is concern that there may be small bowel obstruction developing.

A

The correct answer is McEvedy
This is one approach to an obstructed femoral hernia. It is possible to undertake a small bowel resection through this approach. Recourse to laparotomy may be needed if access is difficult.

How well did you know this?
1
Not at all
2
3
4
5
Perfectly
21
Q

Theme: Surgical access
A. Gridiron
B. Lanz
C. McEvedy
D. Midline abdominal E. Rutherford Morrison F. Battle(abdominal) G. Lower midline
Please select the most appropriate incision for the procedure required. Each option may be used once, more than once or not at all.

A 45 year old woman with end stage renal failure is due to undergo a cadaveric renal transplant. This will be her first transplant.

A

Rutherford Morrison
This is the incision of choice for the extraperitoneal approach to the iliac vessels which will be required for a renal transplant.

How well did you know this?
1
Not at all
2
3
4
5
Perfectly
22
Q

Theme: Surgical access
A. Gridiron
B. Lanz
C. McEvedy
D. Midline abdominal E. Rutherford Morrison F. Battle(abdominal) G. Lower midline
Please select the most appropriate incision for the procedure required. Each option may be used once, more than once or not at all.

A slim 20 year old lady is suffering from appendicitis and requires an appendicectomy.

A

The correct answer is Lanz
Either a Lanz or Gridiron incision will give access for appendicectomy. However, in the case described a Lanz incision will give better cosmesis and can be extended should pelvic surgery be required eg for gynaecological disease.

How well did you know this?
1
Not at all
2
3
4
5
Perfectly
23
Q

Theme: Hernias
A. Littres hernia
B. Richters hernia C. Bochdalek hernia D. Morgagnihernia E. Spigelian hernia F. Lumbar hernia G. Obturator hernia
Please select the type of hernia that most closely matches the description given. Each option may be used once, more than once or not at all.

A 73 year old lady presents with peritonitis and tenderness of the left groin. At operation she has a left femoral hernia with perforation of the anti mesenteric border of ileum associated with the hernia.

A

The correct answer is Richters hernia
When part of the bowel wall is trapped in a hernia such as this it is termed a Richters hernia and may complicate any hernia although femoral and obturator hernias are most typically implicated.

How well did you know this?
1
Not at all
2
3
4
5
Perfectly
24
Q

Theme: Hernias
A. Littres hernia
B. Richters hernia C. Bochdalek hernia D. Morgagnihernia E. Spigelian hernia F. Lumbar hernia G. Obturator hernia
Please select the type of hernia that most closely matches the description given. Each option may be used once, more than once or not at all.

A 22 year old man is operated on for a left inguinal hernia, at operation the sac is opened to reveal a large Meckels diverticulum.

A

Littres hernia
Hernia containing Meckels diverticulum is termed a Littres hernia.

How well did you know this?
1
Not at all
2
3
4
5
Perfectly
25
Q

Theme: Hernias
A. Littres hernia
B. Richters hernia C. Bochdalek hernia D. Morgagnihernia E. Spigelian hernia F. Lumbar hernia G. Obturator hernia
Please select the type of hernia that most closely matches the description given. Each option may be used once, more than once or not at all.

A 45 year old man has recurrent colicky abdominal pain. As part of a series of investigations he undergoes a CT scan and this demonstrates a hernia lateral to the rectus muscle at the level of the arcuate line.

A

The correct answer is Spigelian hernia
This is the site for a spigelian hernia.

How well did you know this?
1
Not at all
2
3
4
5
Perfectly
26
Q

Theme: Groin masses
A. Femoralhernia
B. Lymphadenitis
C. Inguinalhernia
D. Psoas abscess
E. Saphenousvarix
F. Femoral artery aneurysm
G. Metastatic lymphadenopathy H. Lymphoma
I. Falsefemoralarteryaneurysm
What is the likely diagnosis for groin mass described? Each option may be used once, more than once, or not at all.

A 52 year old obese lady reports a painless grape sized mass in her groin area. She has no medical conditions apart from some varicose veins. There is a cough impulse and the mass disappears on lying down.

A

The clinical description of a painless grape-sized mass in the groin area with a cough impulse that disappears when lying down suggests a condition that involves a protrusion or herniation. Given these characteristics, the most likely diagnosis is:

E. Saphenous varix

Explanation:

•	Femoral hernia (A): Often presents as a groin mass and can have a cough impulse, but it tends to be more common in older women and can be painful.
•	Lymphadenitis (B): Typically presents with painful and swollen lymph nodes, often accompanied by signs of infection.
•	Inguinal hernia (C): More common in men, and the description matches this condition; however, the disappearance on lying down is more classically described with saphenous varix.
•	Psoas abscess (D): Usually presents with fever and back pain, not a typical groin mass.
•	Saphenous varix (E): A dilated saphenous vein at the saphenofemoral junction. It presents as a compressible, painless mass that can have a cough impulse and disappears when lying down, fitting the description perfectly.
•	Femoral artery aneurysm (F): Would be pulsatile and generally does not disappear when lying down.
•	Metastatic lymphadenopathy (G): Typically presents as a firm, non-tender, non-reducible mass.
•	Lymphoma (H): Presents with firm, non-tender lymph nodes that do not change with position.
•	False femoral artery aneurysm (I): Would be pulsatile and does not typically disappear when lying down.

Conclusion:

The most likely diagnosis for the described groin mass is a saphenous varix (E). This condition is characterized by a painless, compressible mass with a cough impulse that disappears on lying down, matching the patient’s symptoms.

How well did you know this?
1
Not at all
2
3
4
5
Perfectly
27
Q

Theme: Groin masses
A. Femoralhernia
B. Lymphadenitis
C. Inguinalhernia
D. Psoas abscess
E. Saphenousvarix
F. Femoral artery aneurysm
G. Metastatic lymphadenopathy H. Lymphoma
I. Falsefemoralarteryaneurysm
What is the likely diagnosis for groin mass described? Each option may be used once, more than once, or not at all.

A 32 year old male is noted to have a tender mass in the right groin area. There are also red streaks on the thigh, extending from a small abrasion.

A

The correct answer is Lymphadenitis
The red streaks are along the line of the lymphatics, indicating infection of the lymphatic vessels. Lymphadenitis is infection of the local lymph nodes.

How well did you know this?
1
Not at all
2
3
4
5
Perfectly
28
Q

Theme: Groin masses
A. Femoralhernia
B. Lymphadenitis
C. Inguinalhernia
D. Psoas abscess
E. Saphenousvarix
F. Femoral artery aneurysm
G. Metastatic lymphadenopathy H. Lymphoma
I. Falsefemoralarteryaneurysm
What is the likely diagnosis for groin mass described? Each option may be used once, more than once, or not at all.

A 23 year old male suffering from hepatitis C presents with right groin pain and swelling. On examination there is a large abscess in the groin. Adjacent to this is an expansile swelling. There is no cough impulse.

A

The correct answer is False femoral artery aneurysm
False aneurysms may occur following arterial trauma in IVDU. They may have associated blood borne virus infections and should undergo duplex scanning prior to surgery. False aneurysms do not contain all layers of the arterial wall.

How well did you know this?
1
Not at all
2
3
4
5
Perfectly
29
Q

Theme: Right iliac fossa pain
A. Urinary tract infection B. Appendicitis
C. Mittelschmerz
D. Mesenteric adenitis
E. Crohnsdisease
F. Ulcerativecolitis
G. Meckels diverticulum
Please select the most likely cause for right iliac fossa pain for the scenario given. Each option may be used once, more than once or not at all.

A 17 year old male is admitted with lower abdominal discomfort. He has been suffering from intermittent right iliac fossa pain for the past few months. His past medical history includes a negative colonoscopy and gastroscopy for iron deficiency anaemia. The pain is worse after meals. Inflammatory markers are normal.

A

The correct answer is Meckels diverticulum
This scenario should raise suspicion for Meckels as these may contain ectopic gastric mucosa which may secrete acid with subsequent bleeding and ulceration.

How well did you know this?
1
Not at all
2
3
4
5
Perfectly
30
Q

Theme: Right iliac fossa pain
A. Urinary tract infection B. Appendicitis
C. Mittelschmerz
D. Mesenteric adenitis
E. Crohnsdisease
F. Ulcerativecolitis
G. Meckels diverticulum
Please select the most likely cause for right iliac fossa pain for the scenario given. Each option may be used once, more than once or not at all.

A 14 year old female is admitted with sudden onset right iliac fossa pain. She is otherwise well and on examination has some right iliac fossa tenderness but no guarding. She is afebrile. Urinary dipstick is normal. Her previous menstrual period two weeks ago was normal and pregnancy test is negative.

A

The correct answer is Mittelschmerz
Typical story and timing for mid cycle pain. Mid cycle pain typically occurs because a small amount of fluid is released at the time of ovulation. It will usually resolve over 24-48 hours.

How well did you know this?
1
Not at all
2
3
4
5
Perfectly
31
Q

Theme: Right iliac fossa pain
A. Urinary tract infection B. Appendicitis
C. Mittelschmerz
D. Mesenteric adenitis
E. Crohnsdisease
F. Ulcerativecolitis
G. Meckels diverticulum
Please select the most likely cause for right iliac fossa pain for the scenario given. Each option may be used once, more than once or not at all.

A 21 year old male is admitted with a 3 month history of intermittent right iliac fossa pain. He suffers from episodic diarrhoea and has lost 2 kilos in weight. On examination he has some right iliac fossa tenderness and is febrile.

A

The correct answer is Crohns disease
Weight loss and chronic symptoms coupled with change in bowel habit should raise suspicion for Crohns. The presence of intermittent right iliac fossa pain is far more typical of terminal ileal Crohns disease. Both UC and Crohns may be associated with a low grade pyrexia. The main concern here would be locally perforated Crohns disease with a small associated abscess.

How well did you know this?
1
Not at all
2
3
4
5
Perfectly
32
Q

A 78 year old lady presents with colicky abdominal pain and a tender mass in her groin. On examination there is a small firm mass below and lateral to the pubic tubercle. Which of the following is the most likely underlying diagnosis?
A-Incarcerated inguinal hernia
B-Thrombophlebitis of a saphena varix
C-Incarcerated femoral hernia
D-Incarcerated obturator hernia
E-Deep vein thrombosis

A

Femoral herniae account for <10% of all groin hernias. In the scenario the combination of symptoms of intestinal compromise with a mass in the region of the femoral canal points to femoral hernia as the most likely cause.
Femoral canal
The femoral canal lies at the medial aspect of the femoral sheath. The femoral sheath is a fascial tunnel containing both the femoral artery laterally and femoral vein medially. The canal lies medial to the vein.
Borders of the femoral canal Laterally: Femoral vein Medially: Lacunar ligament Anteriorly : Inguinal ligament Posteriorly: Pectineal ligament

How well did you know this?
1
Not at all
2
3
4
5
Perfectly
33
Q

Which of the following is not a typical feature of acute appendicitis?
Neutrophilia
Profuse vomiting
A n o r e x ia
Low grade pyrexia
Small amounts of protein on urine analysis

A

Profuse vomiting and diarrhoea are rare in early appendicitis
Whilst patients may vomit once or twice, profuse vomiting is unusual, and would fit more with gastroenteritis or an ileus. A trace of protein is not an uncommon occurrence in acute appendicitis. A free lying pelvic appendix may result in localised bladder irritation, with inflammation occurring as a secondary phenomena. This latter feature may result in patients being incorrectly diagnosed as having a urinary tract infection. A urine dipstick test is useful in differentiating between the two conditions.

How well did you know this?
1
Not at all
2
3
4
5
Perfectly
34
Q

An 28 year old man presents with a direct inguinal hernia. A decision is made to perform an open inguinal hernia repair. Which of the following is the best option for abdominal wall reconstruction in this case?
Suture plication of the transversalis fascia using PDS only
Suture plication of the hernial defect with nylon and placement of prolene mesh anterior to external oblique
Suture plication of the hernia defect using nylon and re-enforcing with a sutured repair of the abdominal wall
Sutured repair of the hernial defect with prolene and placement of prolene mesh over the cord structures in the inguinal canal Sutured repair of the hernial defect using nylon and placement of a prolene mesh posterior to the cord structures

A

During an inguinal hernia repair in males the cord structures will always lie anterior to the mesh. In the conventional open repairs the cord structures are mobilised and the mesh placed behind them, with a slit made to allow passage of the cord structures through the deep inguinal ring. Placement of the mesh over the cord structures results in chronic pain and usually a higher risk of recurrence.
Laparoscopic inguinal hernia repair is the procedure of choice for bilateral inguinal hernias.
Types of surgery include:
Onlay mesh repair (Litchenstein style) Inguinal herniorrhaphy
Shouldice repair
Darn repair
Laparoscopic mesh repair
Open mesh repair and laparoscopic repair are the two main procedures in mainstream use. The Shouldice repair is a useful procedure in cases where a mesh repair would be associated with increased risk of infection, e.g. repair of case with strangulated bowel, as it avoids the use of mesh. It is, however, far more technically challenging to perform.
Inguinal hernia surgery
Inguinal hernias occur when the abdominal viscera protrude through the anterior abdominal wall into the inguinal canal. They may be classified as being either direct or indirect. The distinction between these two rests on their relation to Hesselbach’s triangle.
Boundaries of Hesselbach’s Triangle
Medial: Rectus abdominis Lateral: Inferior epigastric vessels Inferior: Inguinal ligament

How well did you know this?
1
Not at all
2
3
4
5
Perfectly
35
Q

Theme: Abdominal closure methods
A. Looped 1/0 PDS (polydiaxone) B. Looped 1/0 silk
C. 1/0 Vicryl (polyglactin)
D. 1/0 Vicryl rapide
E. 2/0 Prolene (Polypropylene) F. Re-inforced 1/0 Nylon
G. Re-inforced 1/0 Silk
H. Application of VAC system without separation film I. ApplicationofVACSystemwithseparationfilm
J. Applicationofa’BogotaBag’
Please select the most appropriate wound closure method (for the deep layer) for the abdominal surgery described.

A 59 year old man with morbid obesity undergoes a laparotomy and Hartmans procedure for perforated sigmoid diverticular disease. At the conclusion of the procedure the abdomen cannot be primarily closed. The Vac system is not available for use.

A

The correct answer is Application of a ‘Bogota Bag’
Application of a Bogota bag is safest as attempted closure will almost certainly fail. Repeat look at 48 hours to determine the best definitive option is needed.

How well did you know this?
1
Not at all
2
3
4
5
Perfectly
36
Q

Theme: Abdominal closure methods
A. Looped 1/0 PDS (polydiaxone) B. Looped 1/0 silk
C. 1/0 Vicryl (polyglactin)
D. 1/0 Vicryl rapide
E. 2/0 Prolene (Polypropylene) F. Re-inforced 1/0 Nylon
G. Re-inforced 1/0 Silk
H. Application of VAC system without separation film I. ApplicationofVACSystemwithseparationfilm
J. Applicationofa’BogotaBag’
Please select the most appropriate wound closure method (for the deep layer) for the abdominal surgery described.

A 73 year old lady undergoes a low anterior resection for carcinoma of the rectum.

A

The correct answer is Looped 1/0 PDS (polydiaxone)
Mass closure obeying Jenkins rule is required and this states that the suture must be 4 times the length of the wound with tissue bites 1cm deep and 1 cm apart.

How well did you know this?
1
Not at all
2
3
4
5
Perfectly
37
Q

Theme: Abdominal closure methods
A. Looped 1/0 PDS (polydiaxone) B. Looped 1/0 silk
C. 1/0 Vicryl (polyglactin)
D. 1/0 Vicryl rapide
E. 2/0 Prolene (Polypropylene) F. Re-inforced 1/0 Nylon
G. Re-inforced 1/0 Silk
H. Application of VAC system without separation film I. ApplicationofVACSystemwithseparationfilm
J. Applicationofa’BogotaBag’
Please select the most appropriate wound closure method (for the deep layer) for the abdominal surgery described.

A 67 year old is returned to theatre after developing a burst abdomen on the ward. She has originally undergone a right hemicolectomy and the SHO who closed the wound had failed to tie the midline suture correctly. The wound edges appear healthy.

A

The correct answer is Re-inforced 1/0 Nylon
Attempt at re-closing the wound is reasonable in which case 1/0 nylon (reinforced with drainage tubing) is often used.
Abdominal wound dehiscence
This is a significant problem facing all surgeons who undertake abdominal surgery on a regular basis. Traditionally, it is said to occur when all layers of an abdominal mass closure fail and the viscera protrude externally (associated with 30% mortality).
It can be subdivided into superficial, in which the skin wound alone fails and complete, implying failure of all layers.

How well did you know this?
1
Not at all
2
3
4
5
Perfectly
38
Q

Theme: Hernias
A. Umbilicalhernia
B. Paraumbilicalhernia C. Morgagnihernia
D. Littres hernia
E. Bochdalek hernia
F. Richtershernia
G. Obturator hernia
Please select the hernia that most closely matches the description given. Each option may be used once, more than once or not at all.

A 1 day old infant is born with severe respiratory compromise. On examination he has a scaphoid abdomen and an absent apex beat.

A

The correct answer is Bochdalek hernia
Theme from 2011 exam
The large hernia may displace the heart although true dextrocardia is not present. The associated pulmonary hypoplasia will compromise lung development.

How well did you know this?
1
Not at all
2
3
4
5
Perfectly
39
Q

Theme: Hernias
A. Umbilicalhernia
B. Paraumbilicalhernia C. Morgagnihernia
D. Littres hernia
E. Bochdalek hernia
F. Richtershernia
G. Obturator hernia
Please select the hernia that most closely matches the description given. Each option may be used once, more than once or not at all.

A 2 month old infant is troubled by recurrent colicky abdominal pain and intermittent intestinal obstruction. On imaging the transverse colon is herniated into the thoracic cavity, through a mid line defect.

A

The correct answer is Morgagni hernia
Morgagni hernia may contain the transverse colon. Unless there is substantial herniation, pulmonary hypoplasia is uncommon. As a result, major respiratory compromise is often absent.

40
Q

Theme: Hernias
A. Umbilicalhernia
B. Paraumbilicalhernia C. Morgagnihernia
D. Littres hernia
E. Bochdalek hernia
F. Richtershernia
G. Obturator hernia
Please select the hernia that most closely matches the description given. Each option may be used once, more than once or not at all.

A 78 year old lady is admitted with small bowel obstruction, on examination she has a distended abdomen and the leg is held semi flexed. She has some groin pain radiating to the ipsilateral knee.

A

The correct answer is Obturator hernia
The groin swelling in obturator hernia is subtle and hard to elicit clinically. There may be pain in the region of sensory distribution of the obturator nerve. The defect is usually repaired from within the abdomen.

41
Q

Theme: Right iliac fossa pain
A. Open Appendicectomy
B. Laparoscopic appendicectomy C. Laparotomy
D. CT Scan
E. Colonoscopy
F. Ultrasound scan abdomen/pelvis G. Active observation
For each scenario please select the most appropriate management option from the list. Each option may be used once, more than once or not at all.

A 21 year old women is admitted with a 48 hour history of worsening right iliac fossa pain. She has been nauseated and vomited twice. On examination she is markedly tender in the right iliac fossa with localised guarding. Vaginal examination is unremarkable. Urine dipstick is negative. Blood tests show a WCC of 13.5 and CRP 70.

A

The correct answer is Laparoscopic appendicectomy
She is likely to have appendicitis. In women of this age there is always diagnostic uncertainty. With a normal vaginal exam laparoscopy would be preferred over USS.

42
Q

Theme: Right iliac fossa pain
A. Open Appendicectomy
B. Laparoscopic appendicectomy C. Laparotomy
D. CT Scan
E. Colonoscopy
F. Ultrasound scan abdomen/pelvis G. Active observation
For each scenario please select the most appropriate management option from the list. Each option may be used once, more than once or not at all.

An 8 year old boy presents with a 4 hour history of right iliac fossa pain with nausea and vomiting. He has been back at school for two days after being kept home with a flu like illness. On examination he is tender in the right iliac fossa, although his abdomen is soft. Temperature is 38.3oc. Blood tests show a CRP of 40 and a WCC of 8.1.

A

Active observation
This is mesenteric adenitis. Note history of flu like illness and temp > 38o c.
The decision as to how to manage this situation is based on the abdominal findings. Patients with localising signs such as guarding or peritonism should undergo surgery.

43
Q

Theme: Right iliac fossa pain
A. Open Appendicectomy
B. Laparoscopic appendicectomy C. Laparotomy
D. CT Scan
E. Colonoscopy
F. Ultrasound scan abdomen/pelvis G. Active observation
For each scenario please select the most appropriate management option from the list. Each option may be used once, more than once or not at al

A 21 year old women presents with right iliac fossa pain. She reports some bloodstained vaginal discharge. She has a HR of 65 bpm.

A

The correct answer is Ultrasound scan abdomen/pelvis
This patient is suspected of having an ectopic pregnancy. She needs an urgent β HCG and USS of the pelvis. If she were haemodynamically unstable then laparotomy would be indicated.

44
Q

Which of the following is not a typical feature of irritable bowel syndrome?
A-A change in the consistency of stools
B-Abdominal pain relieved with defecation
C-A change in frequency of defecation
D-Abdominal bloating
E-Pain at a single fixed site

A

The pain or discomfort of IBS is typically migratory and variable in intensity. Pain at a fixed site is suggestive of malignancy. Abdominal bloating is an extremely common feature.

Irritable bowel syndrome
The diagnosis of irritable bowel syndrome is made according to the ROME III diagnostic criteria which state:
Next question
Recurrent abdominal pain or discomfort at 3 days per month for the past 3 months associated with two or more of the following:
Improvement with defecation.
Onset associated with a change in the frequency of stool. Onset associated with a change in the form of the stool.
Features such as lethargy, nausea, backache and bladder symptoms may also support the diagnosis Red flag features should be inquired about:
Rectal bleeding Unexplained/unintentional weight loss Family history of bowel or ovarian cancer Onset after 60 years of age
Suggested investigations are:
Full blood count
ESR/CRP
Coeliac disease screen (tissue transglutaminase antibodies) Colonoscopy (if worrying symptoms, positive family history) Thyroid function tests
Glucose (ensure not diabetic)
The NICE criteria state that blood tests alone will suffice in people fulfilling the diagnostic criteria. We would point out that luminal colonic studies should be considered early in patients with altered bowel habit referred to hospital and a diagnosis of IBS should still be largely one of exclusion.
Treatment
Usually reduce fibre intake.
Tailored prescriptions of laxatives or loperamide according to clinical picture. Dietary modification (caffeine avoidance, less carbonated drinks).
Consider low dose tricyclic antidepressants if pain is a dominant symptom. Biofeedback may help.

45
Q

Theme: Causes of abdominal pain
A. Acute on chronic mesenteric ischaemia B. Ruptured aortic aneurysm
C. Acute Pancreatitis
D. Acute mesenteric embolus
E. Acuteappendicitis
F. Chronic pancreatitis
G. Mesenteric vein thrombosis
Please select the most likely underlying diagnosis from the list above. Each option may be used once, more than once or not at all.

A 41 year old man is admitted with peritonitis secondary to a perforated appendix. He is treated with a laparoscopic appendicectomy but has a stormy post operative course. He is now developing increasing abdominal pain and has been vomiting. A laparotomy is performed and at operation a large amount of small bowel shows evidence of patchy areas of infarction.

A

The correct answer is Mesenteric vein thrombosis
Mesenteric vein thrombosis may complicate severe intra abdominal sepsis and when it progresses may impair bowel perfusion. The serosa is quite resistant to ischaemia so in this case the appearances are usually patchy.

46
Q

Theme: Causes of abdominal pain
A. Acute on chronic mesenteric ischaemia B. Ruptured aortic aneurysm
C. Acute Pancreatitis
D. Acute mesenteric embolus
E. Acuteappendicitis
F. Chronic pancreatitis
G. Mesenteric vein thrombosis
Please select the most likely underlying diagnosis from the list above. Each option may be used once, more than once or not at all.

A 68 year old man is admitted with abdominal pain and vomiting of 48 hours duration, the pain radiates to his back and he has required a considerable amount of volume replacement. Amylase is 741.

A

The correct answer is Acute Pancreatitis
Although back pain and abdominal pain coupled with haemodynamic compromise may suggest ruptured AAA the 48 hour history and amylase >3 times normal go against this diagnosis.

47
Q

Theme: Causes of abdominal pain
A. Acute on chronic mesenteric ischaemia B. Ruptured aortic aneurysm
C. Acute Pancreatitis
D. Acute mesenteric embolus
E. Acuteappendicitis
F. Chronic pancreatitis
G. Mesenteric vein thrombosis
Please select the most likely underlying diagnosis from the list above. Each option may be used once, more than once or not at all.

A 79 year old lady develops sudden onset of abdominal pain and collapses, she has passed a large amount of diarrhoea. In casualty her pH is 7.35 and WCC is 18.

A

The correct answer is Acute mesenteric embolus
Although mesenteric infarct may raise the lactate the pH may be raised often secondary to vomiting. Mesenteric ischaemia accounts for 1 in 1000 acute surgical admissions. It is primarily caused by arterial embolism resulting in infarction of the colon. It is more likely to occur in areas such as the splenic flexure that are located at the borders of the territory supplied by the superior and inferior mesenteric arteries.

Types
Acute mesenteric embolus (commonest 50%) Sudden onset abdominal pain followed by profuse diarrhoea. May be associated with vomiting. Rapid clinical deterioration.
Serological tests: WCC, lactate, amylase may all be abnormal particularly in established disease. These can be normal in the early phases.
Acute on chronic mesenteric ischaemia
Usually longer prodromal history.
Post prandial abdominal discomfort and weight loss are dominant features. Patients will usually present with an acute on chronic event, but otherwise will tend not to present until mesenteric flow is reduced by greater than 80%.
When acute thrombosis occurs presentation may be as above. In the chronic setting the symptoms will often be those of ischaemic colitis (mucosa is the most sensitive area to this insult).
Mesenteric vein thrombosis
Usually a history over weeks.
Overt abdominal signs and symptoms will not occur until venous thrombosis has reached a stage to compromise arterial inflow. Low flow mesenteric infarction
This occurs in patients with multiple co morbidities in whom mesenteric perfusion is significantly compromised by overuse of inotropes or background cardiovascular compromise.
The end result is that the bowel is not adequately perfused and infarcts occur from the mucosa outwards.
Diagnosis
Serological tests: WCC, lactate, CRP, amylase (can be normal in early disease).
Cornerstone for diagnosis of arterial AND venous mesenteric disease is CT angiography scanning in the arterial phase with thin slices (<5mm). Venous phase contrast is not helpful.
SMA duplex USS is useful in the evaluation of proximal SMA disease in patients with chronic mesenteric ischaemia.
MRI is of limited use due to gut peristalsis and movement artefact.
Management
Overt signs of peritonism: Laparotomy
Mesenteric vein thrombosis: If no peritonism: Medical management with IV heparin
At operation limited resection of frankly necrotic bowel with view to relook laparotomy at 24-48h. In the interim urgent bowel revascularisation via endovascular (preferred) or surgery.
Prognosis
Overall poor. Best outlook is from an acute ischaemia from an embolic event where surgery occurs within 12h. Survival may be 50%. This falls to 30% with treatment delay. The other conditions carry worse survival figures.
Thrombophilia accounts for 60% of cases.

48
Q

Theme: Groin masses
A. Femoralaneurysm B. Lymphadenitis
C. Saphena varix
D. Femoralhernia
E. Indirectinguinalhernia F. Directinguinalhernia G. Psoas abscess
What is the likely diagnosis for the groin mass described? Each option may be used once, more than once or not at all.

A 3 year old boy is referred to the clinic with a scrotal swelling. On examination the mass does not transilluminate and it is impossible to palpate normal cord above it.

A

The correct answer is Indirect inguinal hernia
This is likely to be an indirect hernia. In children these arise from persistent processus vaginalis and require herniotomy.

49
Q

Theme: Groin masses
A. Femoralaneurysm
B. Lymphadenitis
C. Saphena varix
D. Femoralhernia
E. Indirectinguinalhernia
F. Directinguinalhernia
G. Psoas abscess
What is the likely diagnosis for the groin mass described? Each option may be used once, more than once or not at all.

A 3 year old boy is referred to the clinic with a scrotal swelling. On examination the mass does not transilluminate and it is impossible to palpate normal cord above it.

A

The correct answer is Indirect inguinal hernia
This is likely to be an indirect hernia. In children these arise from persistent processus vaginalis and require herniotomy.

50
Q

Theme: Groin masses
A. Femoralaneurysm B. Lymphadenitis
C. Saphena varix
D. Femoralhernia
E. Indirectinguinalhernia F. Directinguinalhernia G. Psoas abscess
What is the likely diagnosis for the groin mass described? Each option may be used once, more than once or not at all.

A 52 year old obese lady reports a painless mass in the groin area. A mass is noted on coughing. It is below and lateral to the pubic tubercle.

A

The correct answer is Femoral hernia
A mass below and lateral to the pubic tubercle is indicative of a femoral hernia.

51
Q

Theme: Groin masses
A. Femoralaneurysm B. Lymphadenitis
C. Saphena varix
D. Femoralhernia
E. Indirectinguinalhernia F. Directinguinalhernia G. Psoas abscess
What is the likely diagnosis for the groin mass described? Each option may be used once, more than once or not at all.

A 21 year old man is admitted with a tender mass in the right groin, fevers and sweats. He is on multiple medical therapy for HIV infection. On examination he has a swelling in his right groin, hip extension exacerbates the pain.

A

Psoas abscess
Psoas abscesses may be either primary or secondary. Primary cases often occur in the immunosuppressed and may occur as a result of haematogenous spread. Secondary cases may complicated intra abdominal diseases such as Crohns. Patients usually present with low back pain and if the abscess is extensive a mass that may be localised to the inguinal region or femoral triangle . Smaller collections may be percutaneously drained. If the collection is larger, or the percutaneous route fails, then surgery (via a retroperitoneal approach) should be performed.

Groin lumps- some key questions
Is there a cough impulse
Is it pulsatile AND is it expansile (to distinguish between false and true aneurysm) Are both testes intra scrotal
Any lesions in the legs such as malignancy or infections (?lymph nodes) Examine the ano rectum as anal cancer may metastasise to the groin
Is the lump soft, small and very superficial (?lipoma)
Scrotal lumps - some key questions
Is the lump entirely intra scrotal Does it transilluminate (?hydrocele) Is there a cough impulse (?hernia)
In most cases a diagnosis can be made clinically. Where it is not clear an ultrasound scan is often the most convenient next investigation.

52
Q

A 56 year old lady is admitted with colicky abdominal pain. A plain x-ray is performed. Which of the following should not show fluid levels on a plain abdominal film?
A-Stomach
B-Jejunum
C-Ileum
D-Caecum
E-Descending colon

A

Fluid levels in the distal colon are nearly always pathological. In general contents of the left colon transit quickly and are seldom held in situ for long periods, the content is also more solid.
Abdominal radiology
Plain abdominal x-rays are often used as a first line investigation in patients with acute abdominal pain. A plain abdominal film may demonstrate free air, evidence of bowel obstruction and possibly other causes of pain (e.g. renal or gallbladder stones).
Investigation of potential visceral perforation is usually best performed by obtaining an erect chest x-ray, as this is a more sensitive investigation for suspected visceral perforation.
Features which are usually abnormal
Large amounts of free air (colonic perforation), smaller volumes seen with more proximal perforations. A positive Riglers sign (gas on both sides of the bowel wall).
Caecal diameter of >8cm
Fluid levels in the colon
Ground glass appearance to film (usually due to large amounts of free fluid). Sentinel loop in patients with inflammation of other organs (e.g. pancreatitis).
Features which should be expected/ or occur without pathology
In Chiladitis sign, a loop of bowel may be interposed between the liver and diaphragm, giving the mistaken impression that free air is present. Following ERCP (and sphincterotomy) air may be identified in the biliary tree.
Free intra abdominal air following laparoscopy / laparotomy, although usually dissipates after 48-72 hours.

53
Q

A 56 year old lady presents with a large bowel obstruction and abdominal distension. Which of the following confirmatory tests should be performed prior to surgery?
A-Abdominal ultrasound scan
B-Barium enema
C-Rectal MRI Scan
D-Endoanal ultrasound scan
E-Gastrograffin enema

A

The confirmatory test that should be performed prior to surgery for a 56-year-old lady presenting with a large bowel obstruction and abdominal distension is Gastrografin enema.

Explanation:

Gastrografin enema is a water-soluble contrast study that can help confirm the diagnosis of a large bowel obstruction by delineating the site and nature of the obstruction. It is preferred over barium enema in acute settings as it is less irritating to the peritoneum in case of perforation.

Other options like abdominal ultrasound, rectal MRI scan, and endoanal ultrasound scan are less effective in identifying and confirming the nature and location of large bowel obstructions in acute settings.

Correct Answer: E-Gastrograffin enema

54
Q

Which of the following does not increase the risk of abdominal wound dehiscence following laparotomy?
A-Jaundice
B-Abdominal compartment syndrome
C-Poorly controlled diabetes mellitus
D-Administration of intravenous steroids
E- Use of Ketamine as an anaesthetic agent

A

Ketamine does not affect healing. All the other situations in the list carry a strong association with poor healing and risk of dehisence.
Abdominal wound dehiscence
Next question
Administration of intravenous steroids
Use of Ketamine as an anaesthetic agent
This is a significant problem facing all surgeons who undertake abdominal surgery on a regular basis. Traditionally, it is said to occur when all layers of an abdominal mass closure fail and the viscera protrude externally (associated with 30% mortality).
It can be subdivided into superficial, in which the skin wound alone fails and complete, implying failure of all layers.
Factors which increase the risk are: * Malnutrition
* Vitamin deficiencies
* Jaundice
* Steroid use
* Major wound contamination (e.g. faecal peritonitis)
* Poor surgical technique (Mass closure technique is the preferred method-Jenkins Rule)
When sudden full dehiscence occurs the management is as follows: * Analgesia
* Intravenous fluids
* Intravenous broad spectrum antibiotics
* Coverage of the wound with saline impregnated gauze (on the ward) * Arrangements made for a return to theatre
Surgical strategy
Correct the underlying cause (e.g. TPN or NG feed if malnourished) Determine the most appropriate strategy for managing the wound
Options
Resuturing of the wound This may be an option if the wound edges are healthy and there is enough tissue for sufficient coverage. Deep tension sutures are traditionally used for this purpose.
Application of a ‘Bogota bag’ This is a clear plastic bag that is cut and sutured to the wound edges and is only a temporary measure to be adopted when the wound cannot be closed and will necessitate a return to theatre for definitive management.
Application of a VAC dressing These can be safely used BUT ONLY if the correct layer is interposed between the suction device and the bowel. Failure to adhere to this absolute rule will almost invariably result in the development of multiple bowel fistulae and create an extremely difficult management problem.
Application of a wound manager This is a clear dressing with removable front. Particularly suitable when some granulation tissue is present over the viscera or where there is a high output bowel fistula present in the dehisced wound.

55
Q

Theme: Causes of diarrhoea
A. Campylobacterjejuniinfection
B. Salmonella gastroenteritis infection C. Crohns disease
D. Ulcerative colitis
E. Irritablebowelsyndrome
F. Ischaemic colitis
G. Laxative abuse
H. Clostridium difficile infection
Please select the most likely cause of diarrhoea for each scenario given. Each option may be used once, more than once or not at all.

A 23 year old lady has suffered from diarrhoea for 8 months, she has also lost 2 Kg in weight. At colonoscopy appearances of melanosis coli are identified and confirmed on biopsy

A

The most likely diagnosis for a 23-year-old lady who has suffered from diarrhea for 8 months, has lost 2 kg in weight, and has colonoscopy appearances of melanosis coli is laxative abuse.

Explanation:

Melanosis coli is a condition characterized by the dark pigmentation of the colon mucosa. It is most commonly associated with chronic use of anthraquinone-containing laxatives, such as senna or cascara. This pigmentation is due to the accumulation of pigment-laden macrophages in the lamina propria of the colon. It is not typically associated with infections, inflammatory bowel diseases, or ischemic colitis.

Correct Answer: G. Laxative abuse

56
Q

Theme: Causes of diarrhoea
A. Campylobacterjejuniinfection
B. Salmonella gastroenteritis infection C. Crohns disease
D. Ulcerative colitis
E. Irritablebowelsyndrome
F. Ischaemic colitis
G. Laxative abuse
H. Clostridium difficile infection
Please select the most likely cause of diarrhoea for each scenario given. Each option may be used once, more than once or not at all.

A 68 year old lady has recently undergone an abdominal aortic aneurysm repair. The operation was performed electively and was uncomplicated. Since surgery she has had repeated episodes of diarrhoea.

A

Ischaemic colitis
The IMA is commonly ligated during and AAA repair and this may then render the left colon relatively ischaemic, thereby causing mesenteric colitis. Treatment is supportive and most cases will settle with conservative management.

57
Q

Theme: Causes of diarrhoea
A. Campylobacterjejuniinfection
B. Salmonella gastroenteritis infection C. Crohns disease
D. Ulcerative colitis
E. Irritablebowelsyndrome
F. Ischaemic colitis
G. Laxative abuse
H. Clostridium difficile infection
Please select the most likely cause of diarrhoea for each scenario given. Each option may be used once, more than once or not at all.

A 23 year old man is admitted to hospital with diarrhoea and severe abdominal pain. He was previously well and his illness has lasted 18 hours.

A

The correct answer is Campylobacter jejuni infection
Severe abdominal pain tends to favour Campylobacter infection.

58
Q

A 6 year old child presents with colicky abdominal pain, vomiting and the passage of red current jelly stool per rectum. On examination the child has a tender abdomen and a palpable mass in the right upper quadrant. Imaging shows an intussusception. Which of the conditions below is least recognised as a precipitant?
A-Inflammation of Payers patches
B-Cystic fibrosis
C-Meckels diverticulum
D-Mesenteric cyst
E-Mucosalpolyps

A

Mesenteric cysts may be associated with intra abdominal catastrophes where these occur they are typically either intestinal volvulus or intestinal infarction. They seldom cause intussusception. Cystic fibrosis may lead to the formation of meconium ileus equivalent and plugs may occasionally serve as the lead points for an intussusception.
Intussusception- Paediatric
Intussusception typically presents with colicky abdominal pain and vomiting. The telescoping of the bowel produces mucosal ischaemia and bleeding may occur resulting in the passage of “red current jelly” stools. Recognised causes include lumenal pathologies such as polyps, lymphadenopathy and diseases such as cystic fibrosis. Idiopathic intussceception of the ileocaecal valve and terminal ileum is the most common variant and typically affects young children and toddlers.
The diagnosis is usually made by abdominal ultrasound investigation. The decision as to the optimal treatment is dictated by the patients physiological status and abdominal signs. In general, children who are unstable with localising peritoneal signs should undergo laparotomy, as should those in whom attempted radiological reduction has failed.
In relatively well children without localising signs attempted pneumatic reduction under fluroscopic guidance is the usual treatment.

59
Q

Which one of the following is least likely to cause malabsorption?
A-Primary biliary cirrhosis
B-Ileo-colic bypass
C-Chronic pancreatitis
D-Whipples disease
E-Hartmans procedure

A

In a Hartmans procedure the sigmoid colon is removed and an end colostomy is fashioned. The bowel remains in continuity and no absorptive ability is lost.

An ileo-colic bypass leaves a redundant loop of small bowel in continuity, where the contents will stagnate and bacterial overgrowth will occur. Therefore this is recognised cause of malabsorption.
Malabsorption
Malabsorption is characterised by diarrhoea, steatorrhoea and weight loss. Causes may be broadly divided into intestinal (e.g. villous atrophy), pancreatic (deficiency of pancreatic enzyme production or secretion) and biliary (deficiency of bile-salts needed for emulsification of fats)
Intestinal causes of malabsorption coeliac disease
Crohn’s disease tropical sprue Whipple’s disease Giardiasis
brush border enzyme deficiencies (e.g. lactase insufficiency)

Pancreatic causes of malabsorption chronic pancreatitis
cystic fibrosis pancreatic cancer
Biliary causes of malabsorption biliary obstruction
primary biliary cirrhosis
Other causes
bacterial overgrowth (e.g. systemic sclerosis, diverticulae, blind loop) short bowel syndrome, lymphoma

60
Q

Theme: Intra abdominal malignancies
A. Metastatic adenocarcinoma of the pancreas B. Metastatic appendiceal carcinoid
C. Metastatic colonic cancer
D. Pseudomyxoma peritonei
E. MALTlymphoma
F. Retroperitonealliposarcoma G. Retroperitonealfibrosis
For the disease given please give the most likely primary disease process. Each option may be used once, more than once or not at all.

A 32 year old man is admitted with a distended tense abdomen. He previously underwent a difficult appendicectomy 1 year previously and was discharged. At laparotomy the abdomen is filled with a gelatinous substance.

A

The correct answer is Pseudomyxoma peritonei
Pseudomyxoma is classically associated with mucin production and the appendix is the commonest source. Pseudomyxoma peritoneii- Curative treatment is peritonectomy (Sugarbaker procedure) and heated intra peritoneal chemotherapy.
Pseudomyxoma Peritonei
Rare mucinous tumour
Most commonly arising from the appendix (other abdominal viscera are also recognised as primary sites) Incidence of 1-2/1,000,000 per year
The disease is characterised by the accumulation of large amounts of mucinous material in the abdominal cavity Treatment
Is usually surgical and consists of cytoreductive surgery (and often peritonectomy c.f Sugarbaker procedure) combined with intra peritoneal chemotherapy with mitomycin C.
Survival is related to the quality of primary treatment and in Sugarbakers own centre 5 year survival rates of 75% have been quoted. Patients with disseminated intraperitoneal malignancy from another source fare far worse.
In selected patients a second look laparotomy is advocated and some practice this routinely.

61
Q

Theme: Intra abdominal malignancies
A. Metastatic adenocarcinoma of the pancreas B. Metastatic appendiceal carcinoid
C. Metastatic colonic cancer
D. Pseudomyxoma peritonei
E. MALTlymphoma
F. Retroperitonealliposarcoma G. Retroperitonealfibrosis
For the disease given please give the most likely primary disease process. Each option may be used once, more than once or not at all.

A 62 year old man is admitted with dull lower back pain and abdominal discomfort. On examination he is hypertensive and a lower abdominal fullness is elicited on examination. An abdominal ultrasound demonstrates hydronephrosis and intravenous urography demonstrated medially displaced ureters. A CT scan shows a periaortic mass.

A

The description of the clinical presentation suggests a condition that involves a mass effect on the retroperitoneal structures, specifically causing hydronephrosis and medially displaced ureters. This is characteristic of retroperitoneal fibrosis.

Key Points:

•	Symptoms: Dull lower back pain, abdominal discomfort, hypertension.
•	Examination: Lower abdominal fullness.
•	Imaging:
•	Ultrasound: Hydronephrosis.
•	Intravenous urography: Medially displaced ureters.
•	CT scan: Periaortic mass.

Diagnosis:

•	Retroperitoneal fibrosis (G): This condition involves the development of fibrotic tissue around the abdominal aorta and other retroperitoneal structures, leading to ureteral obstruction, displacement, and associated symptoms.

Answer:

G. Retroperitoneal fibrosis

62
Q

Theme: Intra abdominal malignancies
A. Metastatic adenocarcinoma of the pancreas B. Metastatic appendiceal carcinoid
C. Metastatic colonic cancer
D. Pseudomyxoma peritonei
E. MALTlymphoma
F. Retroperitonealliposarcoma G. Retroperitonealfibrosis
For the disease given please give the most likely primary disease process. Each option may be used once, more than once or not at all.

A 48 year old lady is admitted with abdominal distension. On examination she is cachectic and has ascites. Her CA19-9 returns highly e le v a t e d .

A

The clinical presentation of a 48-year-old woman with abdominal distension, cachexia, ascites, and a significantly elevated CA19-9 level strongly suggests a diagnosis related to an intra-abdominal malignancy. CA19-9 is a tumor marker commonly associated with pancreatic cancer, but it can also be elevated in other gastrointestinal malignancies.

Key Points:

•	Symptoms: Abdominal distension, cachexia, ascites.
•	Tumor Marker: Elevated CA19-9.

Likely Diagnosis:

•	Metastatic adenocarcinoma of the pancreas: Pancreatic cancer often presents with these symptoms and is commonly associated with elevated CA19-9 levels.

Answer:

A. Metastatic adenocarcinoma of the pancreas

63
Q

Theme: Abdominal pain
A. Acute mesenteric embolus
B. Acute on chronic mesenteric ischaemia C. Mesenteric vein thrombosis
D. Ruptured abdominal aortic aneurysm E. Pancreatitis
F. Appendicitis
G. Acute cholecystitis
Please select the most likely underlying diagnosis from the list above. Each option may be used once, more than once or not at all.

A 72 year old man collapses with sudden onset abdominal pain. He has been suffering from back pain recently and has been taking ibuprofen.

A

The correct answer is Ruptured abdominal aortic aneurysm
Back pain is a common feature with expanding aneurysms and may be miss classified as being of musculoskeletal origin.

64
Q

Theme: Abdominal pain
A. Acute mesenteric embolus
B. Acute on chronic mesenteric ischaemia C. Mesenteric vein thrombosis
D. Ruptured abdominal aortic aneurysm E. Pancreatitis
F. Appendicitis
G. Acute cholecystitis
Please select the most likely underlying diagnosis from the list above. Each option may be used once, more than once or not at all.

A 73 year old women collapses with sudden onset of abdominal pain and the passes a large amount of diarrhoea. On admission she is vomiting repeatedly. She has recently been discharged from hospital following a myocardial infarct but recovered well.

A

The correct answer is Acute mesenteric embolus
Sudden onset of abdominal pain and forceful bowel evacuation are features of acute mesenteric infarct.

65
Q

Theme: Abdominal pain
A. Acute mesenteric embolus
B. Acute on chronic mesenteric ischaemia C. Mesenteric vein thrombosis
D. Ruptured abdominal aortic aneurysm E. Pancreatitis
F. Appendicitis
G. Acute cholecystitis
Please select the most likely underlying diagnosis from the list above. Each option may be used once, more than once or not at all.

A 66 year old man has been suffering from weight loss and develops severe abdominal pain. He is admitted to hospital and undergoes a laparotomy. At operation the entire small bowel is infarcted and only the left colon is viable.

A

The correct answer is Acute on chronic mesenteric ischaemia
This man is likely to have underlying chronic mesenteric vascular disease. Only 15% of emboli will occlude SMA orifice leading to entire small bowel infarct. The background history of weight loss also favours an acute on chronic event. Mesenteric vessel disease
Mesenteric ischaemia accounts for 1 in 1000 acute surgical admissions. It is primarily caused by arterial embolism resulting in infarction of the colon. It is more likely to occur in areas such as the splenic flexure that are located at the borders of the territory supplied by the superior and inferior mesenteric arteries.
Types
Acute mesenteric embolus (commonest 50%)
Sudden onset abdominal pain followed by profuse diarrhoea. May be associated with vomiting.
Rapid clinical deterioration. Serological tests: WCC, lactate, amylase may all be abnormal particularly in established disease. These can be normal in the early phases.

Acute on chronic mesenteric ischaemia
Usually longer prodromal history.
Post prandial abdominal discomfort and weight loss are dominant features. Patients will usually present with an acute on chronic event, but otherwise will tend not to present until mesenteric flow is reduced by greater than 80%.
When acute thrombosis occurs presentation may be as above. In the chronic setting the symptoms will often be those of ischaemic colitis (mucosa is the most sensitive area to this insult).

Mesenteric vein thrombosis
Usually a history over weeks.
Overt abdominal signs and symptoms will not occur until venous thrombosis has reached a stage to compromise arterial inflow.
Thrombophilia accounts for 60% of cases.
Low flow mesenteric infarction
This occurs in patients with multiple co morbidities in whom mesenteric perfusion is significantly compromised by overuse of inotropes or background cardiovascular compromise.
The end result is that the bowel is not adequately perfused and infarcts occur from the mucosa outwards.

Diagnosis
Serological tests: WCC, lactate, CRP, amylase (can be normal in early disease).
Cornerstone for diagnosis of arterial AND venous mesenteric disease is CT angiography scanning in the arterial phase with thin slices (<5mm). Venous phase contrast is not helpful.
SMA duplex USS is useful in the evaluation of proximal SMA disease in patients with chronic mesenteric ischaemia.
MRI is of limited use due to gut peristalsis and movement artefact.
Management
Overt signs of peritonism: Laparotomy
Mesenteric vein thrombosis: If no peritonism: Medical management with IV heparin
At operation limited resection of frankly necrotic bowel with view to relook laparotomy at 24-48h. In the interim urgent bowel revascularisation via endovascular (preferred) or surgery.
Prognosis
Overall poor. Best outlook is from an acute ischaemia from an embolic event where surgery occurs within 12h. Survival may be 50%. This falls to 30% with treatment delay. The other conditions carry worse survival figures.

66
Q

Theme: Acute abdominal pain
A. Appendicitis
B. Henoch Schonlein purpura
C. Diabetes mellitus
D. Intussusception
E. Mittelschmerz
F. Pneumonia
G. Sicklecellcrisis
H. Spontaneousbacterialperitonitis I. Rupureoffollicularcyst
Please select the most likely cause of abdominal pain for the scenario given. Each option may be used once, more than once or not at all.

An 11 month-old girl develops sudden onset abdominal pain. She has a high pitched scream and draws up her legs. Her BP is 90/40 mm/Hg, her pulse 118/min and abdominal examination is normal.

A

The correct answer is Intussusception
Intussusception should be considered in toddlers and infants presenting with screaming attacks. The child often has a history of being unwell for one to three days prior to presentation. The child may pass bloody mucus stool, which is a late sign. Examination of the abdomen is often normal as the sausage mass in the right upper quadrant is difficult to feel.

67
Q

Theme: Acute abdominal pain
A. Appendicitis
B. Henoch Schonlein purpura
C. Diabetes mellitus
D. Intussusception
E. Mittelschmerz
F. Pneumonia
G. Sicklecellcrisis
H. Spontaneousbacterialperitonitis I. Rupureoffollicularcyst
Please select the most likely cause of abdominal pain for the scenario given. Each option may be used once, more than once or not at all.

An 8 year-old West Indian boy presents with periumbilical abdominal pain. He has vomited twice and is refusing fluids. His temperature is 38.1oC and blood tests are as follows: Haemoglobin 8 g/dl, WCC 13 x 109/l, with a neutrophilia.

A

Sickle cell crisis
Sickle cell anaemia is characterised by severe chronic haemolytic anaemia resulting from poorly formed erythrocytes. Painful crises result from vaso-occlusive episodes, which may occur spontaneously or may be precipitated by infection. Consider this diagnosis in all children of appropriate ethnic background.

Appendicitis
History of migratory pain.
Fev er.
A n o r e x ia .
Evidence of right iliac fossa tenderness. Mild pyrexia.
Differential white cell count Pregnancy test C-Reactive protein
A m y la s e
Urine dipstick testing
Appendicectomy
Mesenteric adenitis
Usually recent upper respiratory tract infection.
High fever.
Generalised abdominal discomfort- true localised pain and signs are rare.
Full blood count- may show slightly raised white cell count
Urine dipstick often normal Abdominal ultrasound scan - usually no free fluid
Conservative management- appendicectomy if diagnostic doubt
Mittelschmerz
Only seen in females
Mid cycle pain
Usually occurs two weeks after last menstrual period
Pain usually has a supra-pubic location Usually subsides over a 24-48 hour period.
Full blood count- normal
Urine dipstick- normal
Abdominal and pelvic ultrasound- may show a trace of pelvic free fluid
Manage conservatively if doubt or symptoms fail to settle then laparoscopy
Fitz-Hugh Curtis syndrome
Disseminated infection with Chlamydia. Usually seen in females.
Consists of evidence of pelvic inflammatory disease together with peri- hepatic inflammation and subsequent adhesion formation.
Abdominal ultrasound scan- may show free fluid
High vaginal swabs - may show evidence of sexually transmitted infections
Usually medically managed- doxycycline or a z it h r o m y c in
Abdominal aortic aneurysm (ruptured)
Sudden onset of abdominal pain radiating to the back in older adults (look for risk factors).
Collapse.
May be moribund on arrival in casualty, more stable if contained haematoma. Careful clinical assessment may reveal pulsatile mass.
Patients who are haemodynamically stable should have a CT scan
Unstable patients should undergo immediate surgery (unless it is not in their best interests).
Those with evidence of contained leak on CT should undergo immediate surgery Increasing unruptured aneurysmal size is an indication for urgent surgical intervention (that can wait until the next working day)

Perforated peptic ulcer
Sudden onset of pain (usually epigastric).
Often preceding history of upper abdominal pain.
Soon develop generalised abdominal pain.
On examination may have clinical evidence of peritonitis.
Erect CXR may show free air. A CT scan may be indicated where there is diagnostic doubt
Laparotomy (laparoscopic surgery for perforated peptic ulcers is both safe and feasible in experienced hands)
Intestinal obstruction
Colicky abdominal pain and vomiting (the nature of which depends on the level of the obstruction).
Abdominal distension and constipation (again depending upon site of obstruction).
Features of peritonism may occur where local necrosis of bowel loops is occurring.
A plain abdominal film may help with making the diagnosis. A CT scan may be useful where diagnostic uncertainty exists
In those with a virgin abdomen and lower and earlier threshold for laparotomy should exist than in those who may have adhesional obstruction
Mesenteric infarction
Embolic events present with sudden pain and forceful evacuation.
Acute on chronic events usually have a longer history and previous weight loss. On examination the pain is typically greater than the physical signs would suggest.
Arterial pH and lactate
Arterial phase CT scanning is the most sensitive test
Immediate laparotomy and resection of affected segments, in acute embolic events SMA embolectomy may be needed.

68
Q

Theme: Acute abdominal pain
A. Appendicitis
B. Henoch Schonlein purpura
C. Diabetes mellitus
D. Intussusception
E. Mittelschmerz
F. Pneumonia
G. Sicklecellcrisis
H. Spontaneousbacterialperitonitis I. Rupureoffollicularcyst
Please select the most likely cause of abdominal pain for the scenario given. Each option may be used once, more than once or not at all.

A 15-month-old girl presents with a three day history of periorbital oedema. She is brought to hospital. On examination she has facial oedema and a tender distended abdomen. Her temperature is 39oC and her blood pressure is 90/45 mmHg. There is clinical evidence of poor peripheral perfusion.

A

The correct answer is Spontaneous bacterial peritonitis
The 15-month-old girl is a patient with nephrotic syndrome. Patients with this condition are at risk of septicaemia and peritonitis from Streptococcus pneumoniae, due to the loss of immunoglobulins and opsonins in the urine.

69
Q

Which of the following interventions is most likely to reduce the incidence of intra abdominal adhesions?
A-Peritoneal lavage with cetrimide following elective right hemicolectomy
B-Use of a laparoscopic approach over open surgery
C-Use of talc to coat surgical gloves
D-Performing a Nobles plication of the small bowel
E-Using stapled rather than a hand sewn anastamosis

A

Laparoscopy results in fewer adhesions. When talc was used to coat surgical gloves it was a major cause of adhesion formation and withdrawn for that reason. A Nobles plication is an old fashioned operation which has no place in the prevention of adhesion formation. Use of an anastamotic stapling device will not influence the development of adhesions per se although clearly an anastamotic leak will result in more adhesion formation
Surgical complications
Complications occur in all branches of surgery and require vigilance in their detection. In many cases anticipating the likely complications and appropriate avoidance will minimise their occurrence. For the purposes of the MRCS the important principles to appreciate are:
The anatomical principles that underpin complications
The physiological and biochemical derangements that occur The most appropriate diagnostic modalities to utilise
The principles which underpin their management
This is clearly a very broad area and impossible to cover comprehensively. There is considerable overlap with other topic areas within the website.
Avoiding complications
Some points to hopefully avert complications:
World Health Organisation checklist- now mandatory prior to all operations Prophylactic antibiotics - right dose, right drug, right time.
Assess DVT/ PE risk and ensure adequate prophylaxis
MARK site of surgery
Use tourniquets with caution and with respect for underlying structures
Remember the danger of end arteries and in situations where they occur avoid using adrenaline containing solutions and monopolar diathermy.
Handle tissues with care- devitalised tissue serves as a nidus for infection
Be very wary of the potential for coupling injuries when using diathermy during laparoscopic surgery
The inferior epigastric artery is a favorite target for laparoscopic ports and surgical drains!

Anatomical principles
Understanding the anatomy of a surgical field will allow appreciation of local and systemic complications that may occur. For example, nerve injuries may occur following surgery in specific regions. The table below lists some of the more important nerves to consider and mechanisms of injury

Nerve Mechanism
 Accessory Posterior triangle lymph node biopsy

Sciatic Posterior approach to hip
Common peroneal Legs in Lloyd Davies position
Long thoracic Axillary node clearance
Pelvic autonomic nerves Pelvic cancer surgery
Recurrent laryngeal nerves During thyroid surgery
Hypoglossal nerve During carotid endarterectomy
Ulnar and median nerves During upper limb fracture repairs

These are just a few. The detailed functional sequelae are particularly important and will often be tested. In addition to nerve injuries certain procedures carry risks of visceral or structural injury. Again some particular favorites are given below:
Structure Mechanism
Thoracic duct During thoracic surgery e.g. Pneumonectomy, oesphagectomy
Parathyroid glands During difficult thyroid surgery
Ureters During colonic resections/ gynaecological surgery
Bowel perforation Use of Verres Needle to establish pneumoperitoneum
Bile duct injury Failure to delineate Calots triangle carefully and careless use of diathermy
Facial nerve Always at risk during Parotidectomy
Tail of pancreas When ligating splenic hilum
Testicular vessels During re-do open hernia surgery
Hepatic veins During liver mobilisation
Again many could be predicted from the anatomy of the procedure.
Physiological derangements
A very common complication is bleeding and this is covered under the section of haemorrhagic shock. Another variant is infection either superficial or deep seated. The organisms are covered under microbiology and the features of sepsis covered under shock. Do not forget that immunocompromised and elderly patients may present will atypical physiological parameters.

Selected physiological and biochemical issues are given below:
Complication Physiological/ Biochemical Problem
Arrhythmias following cardiac Susceptibility to hypokalaemia (K+ <4.0 in cardiac patients) surgery
Neurosurgical electrolyte disturbance SIADH following cranial surgery causing hyponatraemia
Ileus following gastrointestinal Fluid sequestration and loss of electrolytes surgery
Pulmonary oedema following Loss of lung volume makes these patients very sensitive to fluid overload pneumonectomy
Anastamotic leak Generalised sepsis causing mediastinitis or peritonitis depending on site of leak
Myocardial infarct May follow any type of surgery and in addition to direct cardiac effects the decreased cardiac output may well compromise grafts etc.

70
Q

Theme: Abdominal pain
A. Appendicitis
B. Threatened miscarriage
C. Ectopic pregnancy
D. Irritablebowelsyndrome E. Mittelschmerz
F. Pelvic inflammatory disease G. Adnexialtorsion
H. Endometriosis
I. Degeneratingfibroid
Please select the most likely cause of abdominal pain for the clinical scenario given. Each option may be used once, more than once or not at all.

An 18 year-old girl presents to the Emergency Department with sudden onset sharp, tearing pelvic pain associated with a small amount of vaginal bleeding. She also complains of shoulder tip pain. On examination she is hypotensive, tachycardic and has marked cervical
e x c it a t io n .

A

The correct answer is Ectopic pregnancy
The history of tearing pain and haemodynamic compromise in a women of child bearing years should prompt a diagnosis of ectopic pregnancy.

71
Q

Theme: Abdominal pain
A. Appendicitis
B. Threatened miscarriage
C. Ectopic pregnancy
D. Irritablebowelsyndrome E. Mittelschmerz
F. Pelvic inflammatory disease G. Adnexialtorsion
H. Endometriosis
I. Degeneratingfibroid
Please select the most likely cause of abdominal pain for the clinical scenario given. Each option may be used once, more than once or not at all.

A 25 year-old lady presents to her GP complaining of a two day history of right upper quadrant pain, fever and a white vaginal discharge. She has seen the GP twice in 12 weeks complaining of pelvic pain and dyspareunia.

A

The correct answer is Pelvic inflammatory disease
The most likely diagnosis is pelvic inflammatory disease. Right upper quadrant pain occurs as part of the Fitz Hugh Curtis syndrome in which peri hepatic inflammation occurs.

72
Q

Theme: Abdominal pain
A. Appendicitis
B. Threatened miscarriage
C. Ectopic pregnancy
D. Irritablebowelsyndrome E. Mittelschmerz
F. Pelvic inflammatory disease G. Adnexialtorsion
H. Endometriosis
I. Degeneratingfibroid
Please select the most likely cause of abdominal pain for the clinical scenario given. Each option may be used once, more than once or not at all.

A 16 year old female presents to the emergency department with a 12 hour history of pelvic discomfort. She is otherwise well and her last normal menstrual period was 2 weeks ago. On examination she has a soft abdomen with some mild supra pubic discomfort.

A

The correct answer is Mittelschmerz
Mid cycle pain is very common and is due to the small amount of fluid released during ovulation. Inflammatory markers are usually normal and the pain typically subsides over the next 24-48 hours.

73
Q

Which of the following statements relating to a burst abdomen is false?
Is seen in 1-2% of modern laparotomies
Is more common in faecal peritonitis
Is less common when a ‘mass closure’ technique is used
When it does occur is most common at 15 days
Is similar in incidence regardless of whether 1/0 polydiaxone or 1/0 polypropylene are used

A

Correct Answer:

When it does occur is most common at 15 days

Explanation:

A burst abdomen, also known as wound dehiscence, typically occurs within the first week after surgery, usually around 7-10 days postoperatively. This timeline is crucial as the wound strength is at its lowest during this period due to the inflammatory phase of wound healing.

Here is an analysis of the other statements:

•	Is seen in 1-2% of modern laparotomies: True. The incidence of burst abdomen is around 1-2% in modern surgical practice.
•	Is more common in faecal peritonitis: True. Faecal peritonitis increases the risk of wound dehiscence due to infection and increased intra-abdominal pressure.
•	Is less common when a ‘mass closure’ technique is used: True. Mass closure techniques, which involve taking large bites of tissue including fascia, are associated with a lower incidence of burst abdomen compared to layered closure techniques.
•	Is similar in incidence regardless of whether 1/0 polydiaxone or 1/0 polypropylene are used: True. Studies have shown that the type of suture material (whether absorbable like polydiaxone or non-absorbable like polypropylene) does not significantly affect the incidence of wound dehiscence.

Thus, the incorrect statement is that wound dehiscence is most common at 15 days, which is not accurate as it typically occurs earlier in the postoperative period.

74
Q

Which of the following statements about diarrhoea is false?
Nocturnal diarrhoea is uncommon in irritable bowel syndrome
World Health Organisation definition of diarrhoea is greater than 3 episodes of loose or watery stool a day
Pancreatic disease causes osmotic diarrhoea
Vitamin C deficiency causes diarrhoea
The World Health Organisation definition of chronic diarrhoea is greater than 14 days of diarrhoea

A

Vitamin C toxicity causes osmotic diarrhoea.
Diarrhoea
World Health Organisation definitions
Diarrhoea: > 3 loose or watery stool per day Acute diarrhoea < 14 days
Chronic diarrhoea > 14 days
Acute Diarrhoea Gastroenteritis
Diverticulitis Antibiotic therapy
Constipation causing overflow
Chronic Diarrhoea
May be accompanied by abdominal pain or nausea/vomiting Classically causes left lower quadrant pain, diarrhoea and fever
More common with broad spectrum antibiotics Clostridium difficile is also seen with antibiotic use
A history of alternating diarrhoea and constipation may be given May lead to faecal incontinence in the elderly
Irritable bowel syndrome
Extremely common. The most consistent features are abdominal pain, bloating and change in bowel habit. Patients may be divided into those with diarrhoea predominant IBS and those with constipation predominant IBS.
Features such as lethargy, nausea, backache and bladder symptoms may also be present
Ulcerative colitis
Crohn’s disease
Colorectal cancer
Coeliac disease
Bloody diarrhoea may be seen. Crampy abdominal pain and weight loss are also common. Faecal urgency and tenesmus may occur
Crampy abdominal pains and diarrhoea. Bloody diarrhoea less common than in ulcerative colitis. Other features include malabsorption, mouth ulcers perianal disease and intestinal obstruction
Symptoms depend on the site of the lesion but include diarrhoea, rectal bleeding, anaemia and constitutional symptoms e.g. Weight loss and anorexia
In children may present with failure to thrive, diarrhoea and abdominal distension
In adults lethargy, anaemia, diarrhoea and weight loss are seen. Other autoimmune conditions may coexist

75
Q

A 40 year old man presents with a long standing inguinal hernia. On examination he has a small, direct inguinal hernia. He inquires as to the risk of strangulation over the next twelve months should he decide not to undergo surgery. Which of the following most closely matches the likely risk of strangulation over the next 12 months?
50%
40%
25%
15%
<5%

A

The annual probability of strangulation is up to 3% and is more common in indirect hernias. Elective repair poses few risks. However, emergency repair is associated with increased mortality, particularly in the elderly.
Inguinal hernia surgery
Inguinal hernias occur when the abdominal viscera protrude through the anterior abdominal wall into the inguinal canal. They may be classified as being either direct or indirect. The distinction between these two rests on their relation to Hesselbach’s triangle.
Boundaries of Hesselbach’s Triangle
Medial: Rectus abdominis Lateral: Inferior epigastric vessels Inferior: Inguinal ligament

76
Q

A 60 year old women has fully recovered from an attack of pancreatitis. Over the following 12 months she develops episodic epigastric discomfort. Un upper GI endoscopy shows gastric varices only. An abdominal CT scan demonstrates a splenic vein thrombosis. What is the treatment of choice?
Splenectomy
Insertion of transjugular porto-systemic shunt
Surgical bypass of the splenic vein
Gastrectomy
Stapling of the gastro-oesophgeal junction

A

Thrombosis of the splenic vein may complicate pancreatitis, pancreatic carcinoma, iatrogenic trauma and hypercoagulable diseases. Diagnosis is made by CT angiography.

The treatment of choice for a patient with gastric varices secondary to splenic vein thrombosis is:

Splenectomy

Explanation:

•	Splenic vein thrombosis can lead to isolated gastric varices due to increased pressure in the short gastric veins, which drain into the splenic vein. This can cause episodic epigastric discomfort and the development of varices.
•	Splenectomy is the treatment of choice because it removes the source of increased pressure, thereby alleviating the gastric varices and preventing further episodes of bleeding.

Other options are less appropriate in this context:

•	Insertion of a transjugular intrahepatic portosystemic shunt (TIPS) is typically used for portal hypertension involving the portal vein and is not specifically indicated for isolated splenic vein thrombosis.
•	Surgical bypass of the splenic vein is not a standard treatment for this condition.
•	Gastrectomy is an overly aggressive approach for managing varices caused by splenic vein thrombosis.
•	Stapling of the gastro-oesophageal junction is not a treatment for gastric varices and does not address the underlying cause of the varices in this scenario.

Therefore, the correct answer is Splenectomy.

77
Q

Theme: Surgical signs
A. Rovsing’s sign
B. Boas’ sign
C. Psoas stretch sign
D. Cullen’s sign
E. Grey-Turner’ssign
F. Murphy’ssign
G. None of the above
Please match the clinical sign to the clinical scenario described. Each option may be used once, more than once or not at all.

Acute retrocaecal appendicitis is indicated when the right thigh is passively extended with the patient lying on their side with their knees extended.

A

The clinical sign that matches the described scenario of acute retrocaecal appendicitis indicated when the right thigh is passively extended with the patient lying on their side with their knees extended is:

C. Psoas stretch sign

Explanation:

•	Psoas stretch sign (also known as the psoas sign) is used to identify inflammation of the appendix, particularly when it is retrocaecal. This sign is elicited by passively extending the right thigh with the patient lying on their left side. Pain elicited during this maneuver suggests irritation of the psoas muscle by the inflamed appendix, indicating retrocaecal appendicitis.

Therefore, the correct answer is C. Psoas stretch sign.

78
Q

Theme: Surgical signs
A. Rovsing’s sign
B. Boas’ sign
C. Psoas stretch sign
D. Cullen’s sign
E. Grey-Turner’ssign
F. Murphy’ssign
G. None of the above
Please match the clinical sign to the clinical scenario described. Each option may be used once, more than once or not at all.

In acute pancreatitis there is bruising in the flanks.

A

The correct answer is Grey-Turner’s sign
Grey-Turners sign occurs in patients with severe haemorrhagic pancreatitis. In this situation the major vessels surrounding the pancreas bleed. The pancreatitis process also results in local fat destruction, this results in blood tracking in the tissue planes of the retroperitoneum and appearing as flank bruising.

79
Q

Theme: Surgical signs
A. Rovsing’s sign
B. Boas’ sign
C. Psoas stretch sign
D. Cullen’s sign
E. Grey-Turner’s sign
F. Murphy’ssign
G. None of the above
Please match the clinical sign to the clinical scenario described. Each option may be used once, more than once or not at all.

In cholecystitis there is pain/catch of breath elicited on palpation of the right hypochondrium during inspiration.

A

Murphy’s sign
Invariably present when patients are assessed in the emergency department! This sign occurs because the inflamed gallbladder irritates the parietal peritoneum in this manoeuvre.

80
Q

Which of the following is commonest cause of acute abdominal pain in acute unselected surgical take’?
Non specific abdominal pain
Biliary colic
Acute appendicitis
Ureteric colic
Pancreatitis

A

Non specific abdominal pain is a commonly recorded diagnosis for patients presenting with acute abdominal pain. Following careful diagnostic work up, a proportion of patients may be identified with disorders such as coeliac disease and the diagnosis of non specific abdominal pain should not be used lightly.
Acute abdominal pain
Acute abdominal pain is a common cause of admission to hospital. The relative proportions of conditions presenting with abdominal pain is given below:
Non specific abdominal pain (35%) Appendicitis (17%)
Intestinal obstruction (15%) Urological disease (6%)
Gallstone disease (5%)
Colonic diverticular disease (4%) Abdominal trauma (3%) Perforated peptic ulcer (3%) Pancreatitis (2%)
(Data derived from Irvin T. Br. J. Surg 1989 76:1121-1125)
Non specific abdominal pain should really be a diagnosis of exclusion and if care is taken in excluding organic disease the proportion of cases labeled such should decline. It should also be appreciated that a proportion of patients may have an underlying medical cause for their symptoms such as pneumonia or diabetic ketoacidosis.
Key points in management
Early administration of adequate analgesia (including opiates).
Abdominal ultrasound is safe, non invasive and cheap and yields significantly more information than plain radiology. However, plain radiology is still the main test for suspected perforated viscus, especially out of hours.
In up to 50% cases with perforated peptic ulcer, the plain x-rays may show no evidence of free air. If clinical signs suggest otherwise, then a CT scan may be a more accurate investigation, if plain films are normal.
Plain film radiology usually cannot detect <1mm free air, and is 33% sensitive for detection of 1-13mm pockets of free air (Stoker et al. Radiology 2009 253: 31-46).
Think of strangulated intestine when there is fever, raised white cell count, tachycardia and peritonism.
In suspected large bowel obstruction a key investigation is either a water soluble contrast enema or CT scan.
Where need for surgery is difficult to define and imaging is inconclusive the use of laparoscopy as a definitive diagnostic test is both safe and sensible.

81
Q

A 72 year old obese man undergoes and emergency repair of a ruptured abdominal aortic aneurysm. The wound is closed with an onlay prolene mesh to augment the closure. Post operatively he is taken to the intensive care unit. Over the following twenty four hours his nasogastric aspirates increase, his urine output falls and he has a metabolic acidosis. What is the most likely underlying cause?

Colonic ischaemia
Abdominal compartment syndrome
Peritonitis
Reactionary haemorrhage
Aorto-duodenal fistula

A

Obese patients with ileus following major abdominal surgery are at increased risk of intra abdominal compartment syndrome.The risk is increased by the use of prosthetic meshes, which some surgeons favor following a major vascular case as they may reduce the incidence of incisional hernia. They prevent abdominal distension and may increase the risk of intra abdominal hypertension in the short term. Although colonic ischaemia may occur following major aortic surgery it would not typically present in this way.
Abdominal compartment syndrome
Background
Intra-abdominal pressure is the steady state pressure concealed within the abdominal cavity. In critically ill adults the normal intra abdominal pressure = 5-7mmHg
Intra abdominal hypertension has pressures of 12-25mmHg
Changes >15mmHg are associated with microvascular hypoperfusion
Abdominal compartment syndrome is defined as sustained intra abdominal pressure >20mmHg coupled with new organ dysfunction / failure
It may occur either primarily without previous surgical intervention e.g. Following intestinal ischaemia or secondarily following a surgical procedure
Diagnosis is typically made by transvesical pressure measurements coupled with an index of clinical suspicion.
Management
Once the diagnosis is made non operative measures should be instituted including: Gastric decompression
Improve abdominal wall compliance e.g. muscle relaxants/ sedation Drain abdominal fluid collections.
Consider fluid restriction/ diuretics if clinically indicated.
In those whom non operative treatment is failing; the correct treatment is laparotomy and laparostomy. Options for laparostomy are many although the Bogota bag or VAC techniques are the most widely practised. Re-look laparotomy and attempts at delayed closure will follow in due course.

82
Q

Theme: Surgical drains

A. Redivac suction drain
B. Corrugated drain
C. Wallace Robinson drain
D. Penrose tubing
E. Latex T Tube drain
F. Silastic T Tube drain

Please select the most appropriate surgical drainage system for the indication given. Each option may be used once, more than once or not at all.

A 56 year old lady undergoes and open cholecystectomy and exploration of common bile duct. The bile duct is closed over a drain.

A

The correct answer is Latex T Tube drain
Latex is used for this indication as it will encourage track formation.

83
Q

Theme: Surgical drains
A. Redivac suction drain
B. Corrugated drain
C. Wallace Robinson drain
D. Penrose tubing
E. Latex T Tube drain
F. Silastic T Tube drain
Please select the most appropriate surgical drainage system for the indication given. Each option may be used once, more than once or not at all.

A 48 year old lady undergoes a mastectomy and axillary node clearance for an invasive ductal cancer of the breast with lymph node metastasis.

A

The correct answer is Redivac suction drain
Suction drains are commonly used following mastectomy and axillary surgery to prevent haematoma formation. Not all surgeons routinely drain the axilla.

84
Q

Theme: Surgical drains
A. Redivac suction drain B. Corrugated drain
C. Wallace Robinson drain D. Penrose tubing
E. Latex T Tube drain F. Silastic T Tube drain
Please select the most appropriate surgical drainage system for the indication given. Each option may be used once, more than once or not at all.

A 75 year old man undergoes a hartmans procedure for sigmoid diverticular disease with pericolic abscess and colovesical fistula.

A

Wallace Robinson drain
These tube drains are often used in abdominal surgery to drain abscess cavities. Debate might occur around the use of low pressure vs no suction in this setting so this option is deliberately omitted. Surgical drains
Drains are inserted in many surgical procedures and are of many types.
As a broad rule they can be divided into those using suction and those which do not.
The diameter of the drain will depend upon the substance being drained, for example smaller lumen drain for pneumothoraces vs haemothorax.
Drains can be associated with complications and these begin with insertion when there may be iatrogenic damage. When in situ they serve as a route for infections. In some specific situations they may cause other complications, for example suction drains left in contact with bowel for long periods may carry a risk of inducing fistulation.
Drains should be inserted for a defined purpose and removed once the need has passed.
A brief overview of types of drain and sites is given below
CNS Low suction drain or free drainage systems may be used for situations such as drainage of sub dural haematomas.
CVS
Orthopaedics and trauma
In this setting drains are usually used to prevent haematoma formation (with associated risk of infection). Some orthopaedic drains may also be specially adapted to allow the drained blood to be auto transfused.
Gastro-intestinal surgery
Surgeons often place abdominal drains either to prevent or drain abscesses, or to turn an anticipated complication into one that can be easily controlled such as a bile leak following cholecystectomy. The type of drain used will depend upon the indication.
Following cardiothoracic procedures of thoracic trauma underwater seal drains are often placed. These should be carefully secured. When an air leak is present they may be placed on suction whilst the air leak settles
Drain types Type of drain
Features
R e d iv a c
Suction type of drain
Closed drainage system
High pressure vacuum system
Low pressure drainage systems
Consist of small systems such as the lantern style drain that may be used for short term drainage of small wounds and cavities
Larger systems are sometimes used following abdominal surgery, they have a lower pressure than the redivac system, which decreases the risks of fistulation
May be emptied and re-pressurised
Latex tube drains
May be shaped (e.g. T Tube) or straight
Usually used in non pressurised systems and act as sump drains
Most often used when it is desirable to generate fibrosis along the drain track (e.g. following exploration of the CBD)

Chest drains May be large or small diameter (depending on the indication) Connected to underwater seal system to ensure one way flow of air
Corrugated drain Thin, wide sheet of plastic, usually soft Contains corrugations, along which fluids can track

85
Q

What is the commonest site in the abdomen for fluid to collect following a perforated appendix?
P e lv is
Hepatorenal pouch
Between small bowel loops
Right iliac fossa
Lesser sac

A

Following perforated appendicitis fluid is most likely to accumulate in the pelvis. Fluid may accumulate in the hepatorenal pouch although this is less common. Gravity favors the pelvis as the site of most collections. The incidence of these is higher with laparoscopic rather than open surgery.

86
Q

Theme: Right iliac fossa pathology
A. Appendicitis
B. Mesenteric adenitis
C. Inflammatory bowel disease D. Irritablebowelsyndrome E. Mesenteric cyst
F. Campylobacter infection
G. Appendix abscess
Please select the most likely diagnosis for the scenario given. Each option may be used once, more than once or not at all.

An 8 year old boy is examined by his doctor as part of a routine clinical examination. The doctor notices a smooth swelling in the right iliac fossa. It is mobile and the patient is otherwise well.

A

The correct answer is Mesenteric cyst
Theme from April 2012 Exam
Mesenteric cysts are often smooth. Imaging with ultrasound and CT is usually sufficient. Although rare, they most often occur in young children (up to 30% present before the age of 15). Many are asymptomatic and discovered incidentally. Acute presentations are recognised and may occur following cyst torsion, infarction or rupture. Most cysts will be surgically resected.

87
Q

Theme: Right iliac fossa pathology
A. Appendicitis
B. Mesenteric adenitis
C. Inflammatory bowel disease
D. Irritable bowel syndrome
E. Mesenteric cyst
F. Campylobacter infection
G. Appendix abscess
Please select the most likely diagnosis for the scenario given. Each option may be used once, more than once or not at all.

An 8 year old boy presents with abdominal pain,a twelve hour history of vomiting, a fever of 38.3 oC and four day history of diarrhoea. His abdominal pain has been present for the past week.

A

The correct answer is Appendix abscess
The high fever and diarrhoea together with vomiting all point to a pelvic abscess. The presence of pelvic pus is highly irritant to the rectum, and many patients in this situation will complain of diarrhoea.

88
Q

Theme: Right iliac fossa pathology
A. Appendicitis
B. Mesenteric adenitis
C. Inflammatory bowel disease D. Irritablebowelsyndrome E. Mesenteric cyst
F. Campylobacter infection
G. Appendix abscess
Please select the most likely diagnosis for the scenario given. Each option may be used once, more than once or not at all.

A 7 year old boy presents with a three day history of right iliac fossa pain and fever. On examination he has a temperature of 39.9o C. His abdomen is soft and mildly tender in the right iliac fossa.

A

The correct answer is Mesenteric adenitis
High fever and mild abdominal signs in a younger child should raise suspicion for mesenteric adenitis. The condition may mimic appendicitis and many may require surgery.

89
Q

A 56 year old man undergoes a difficult splenectomy and is left with a pancreatic fistula. There are ongoing problems with very high fistula output. Which of the following agents may be administered to reduce the fistula output?

Metoclopramide
Erthyromycin
Octreotide
Loperamide
Omeprazole

A

Octreotide is a useful agent in reducing the output from pancreatic fistulae. Prokinetic agents will increase fistula output and should be avoided.
Fistulas
A fistula is defined as an abnormal connection between two epithelial surfaces.
There are many types ranging from Branchial fistulae in the neck to entero-cutaneous fistulae abdominally.
In general surgical practice the abdominal cavity generates the majority and most of these arise from diverticular disease and Crohn’s. As a general rule all fistulae will resolve spontaneously as long as there is no distal obstruction. This is particularly true of intestinal fistulae.
The four types of fistulae are:
Enterocutaneous
These link the intestine to the skin. They may be high (>500ml) or low output (<250ml) depending upon source. Duodenal /jejunal fistulae will tend to produce high volume, electrolyte rich secretions which can lead to severe excoriation of the skin. Colo-cutaneous fistulae will tend to leak faeculent material. Both fistulae may result from the spontaneous rupture of an abscess cavity onto the skin (such as following perianal abscess drainage) or may occur as a result of iatrogenic input. In some cases it may even be surgically desirable e.g. mucous fistula following sub total colectomy for colitis.
Suspect if there is excess fluid in the drain.
Enteroenteric or Enterocolic
This is a fistula that involves the large or small intestine. They may originate in a similar manner to enterocutaneous fistulae. A particular problem with this fistula type is that bacterial overgrowth may precipitate malabsorption syndromes. This may be particularly serious in inflammatory bowel disease.
Enterovaginal
Aetiology as above.
Enterovesicular
This type of fistula goes to the bladder. These fistulas may result in frequent urinary tract infections, or the passage of gas from the urethra during urination.

Management
Some rules relating to fistula management:
They will heal provided there is no underlying inflammatory bowel disease and no distal obstruction, so conservative measures may be the best option
Where there is skin involvement, protect the overlying skin, often using a well fitted stoma bag- skin damage is difficult to treat
A high output fistula may be rendered more easily managed by the use of octreotide, this will tend to reduce the volume of pancreatic secretions.
Nutritional complications are common especially with high fistula (e.g. high jejunal or duodenal) these may necessitate the use of TPN to provide nutritional support together with the concomitant use of octreotide to reduce volume and protect skin.
When managing perianal fistulae surgeons should avoid probing the fistula where acute inflammation is present, this almost always worsens outcomes.
When perianal fistulae occur secondary to Crohn’s disease the best management option is often to drain acute sepsis and maintain that drainage through the judicious use of setons whilst medical management is implemented.
Always attempt to delineate the fistula anatomy, for abscesses and fistulae that have an intra abdominal source the use of barium and CT studies should show a track. For perianal fistulae surgeons should recall Goodsall’s rule in relation to internal and external openings.

90
Q

Theme: Inguinal hernia management
A. Bassinirepair
B. Inguinal herniotomy
C. Lichtenstein repair
D. Laparoscopic hernia repair
E. Shouldice repair
F. McVey repair
For the herniae described please select the most appropriate procedure from the list. Each option may be used once, more than once or not at all.

A 11 month old child presents with intermittent groin swelling, it has a cough impulse and is easily reducible.

A

The correct answer is Inguinal herniotomy
Infants usually suffer from a patent processus vaginalis (a congential problem). As a result a simple herniotomy is all that is required. A mesh is not required as there is not specific muscle weakness.

91
Q

Theme: Inguinal hernia management
A. Bassinirepair
B. Inguinalherniotomy
C. Lichtenstein repair
D. Laparoscopic hernia repair E. Shouldicerepair
F. McVey repair
For the herniae described please select the most appropriate procedure from the list. Each option may be used once, more than once or not at all.

A 25 year old builder presents with a reducible swelling in the right groin, it is becoming larger and has not been operated on previously.

A

The correct answer is Lichtenstein repair
An open Lichtenstein repair using mesh is appropriate. There is a 0.77% recurrence rate with this technique. A Shouldice repair is an acceptable alternative if the surgeon is experienced

92
Q

Theme: Inguinal hernia management
A. Bassinirepair
B. Inguinalherniotomy
C. Lichtenstein repair
D. Laparoscopic hernia repair E. Shouldicerepair
F. McVey repair
For the herniae described please select the most appropriate procedure from the list. Each option may be used once, more than once or not at all.

A 28 year old man presents with a recurrent inguinal hernia on the left side of his abdomen and a newly diagnosed inguinal hernia on the right side.

A

The correct answer is Laparoscopic hernia repair
Laparoscopic hernia repairs are specifically indicated where there are bilateral hernias or recurrence of a previous open repair.
Next question
Inguinal hernia surgery
Inguinal hernias occur when the abdominal viscera protrude through the anterior abdominal wall into the inguinal canal. They may be classified as being either direct or indirect. The distinction between these two rests on their relation to Hesselbach’s triangle.
Boundaries of Hesselbach’s Triangle
Medial: Rectus abdominis Lateral: Inferior epigastric vessels Inferior: Inguinal ligament

93
Q

Theme: Surgical access
A. Kocher’s
B. Lanz
C. Rooftop
D. Pfannenstiel’s E. Midline
F. Paramedian incision G. Mcevedy
Please select the most appropriate incision for the procedure described. Each option may be used once, more than once or not at all.

A 19 year old girl who is 39 weeks pregnant goes into labour. The labour is prolonged and she is found to have an undiagnosed breech baby.

A

The correct answer is Pfannenstiel’s
This patient needs an emergency cesarean section.

94
Q

Theme: Surgical access
A. Kocher’s
B. Lanz
C. Rooftop
D. Pfannenstiel’s
E. Midline
F. Paramedian incision
G. McEvedy
Please select the most appropriate incision for the procedure described. Each option may be used once, more than once or not at all.

A 49 year old woman presents with jaundice and abdominal pain. She is haemodynamically unstable. An USS shows a dilated common bile duct and gallstones in the gallbladder.

A

Correct Answer:

A. Kocher’s

Explanation:

In this scenario, the patient likely requires an urgent cholecystectomy and possible exploration of the common bile duct due to gallstones causing jaundice and a dilated common bile duct.

•	Kocher’s incision: This is a right subcostal incision that provides excellent access to the gallbladder, common bile duct, and liver. It is commonly used for open cholecystectomy and biliary surgeries.

Other incisions are less appropriate for this particular situation:

•	Lanz: Typically used for appendectomies.
•	Rooftop: Commonly used for extensive upper abdominal surgeries, such as pancreaticoduodenectomy.
•	Pfannenstiel’s: A low transverse incision used for pelvic surgeries, particularly gynecological and obstetric procedures.
•	Midline: Provides good access to most abdominal structures but is not specifically tailored for biliary surgery.
•	Paramedian incision: Less commonly used nowadays; offers good access to certain abdominal organs but not typically for biliary surgery.
•	McEvedy: Used for femoral hernia repairs.

Therefore, the most appropriate incision for this procedure is Kocher’s.

95
Q

Theme: Surgical access
A. Kocher’s
B. Lanz
C. Rooftop
D. Pfannenstiel’s E. Midline
F. Paramedian incision G. Mcevedy
Please select the most appropriate incision for the procedure described. Each option may be used once, more than once or not at all.

A 42 year old man with history of alcohol abuse is diagnosed with pancreatic cancer and requires a Whipples resection.

A

The correct answer is Rooftop
A pancreatectomy is usually performed through a roof top incision. This provides excellent access to the upper abdomen.
Abdominal incisions
Midline incision
Commonest approach to the abdomen
Structures divided: linea alba, transversalis fascia, extraperitoneal fat, peritoneum (avoid falciform ligament above the umbilicus)
Bladder can be accessed via an extraperitoneal approach through the space of Retzius
Paramedian incision
Parallel to the midline (about 3-4cm)
Structures divided/retracted: anterior rectus sheath, rectus (retracted), posterior rectus sheath, transversalis fascia, extraperitoneal fat, peritoneum
Incision is closed in layers
Battle Similar location to paramedian but rectus displaced medially (and thus denervated)

Kocher’s
Lanz
Gridiron
Gable
Pfannenstiel’s
McEvedy’s
Rutherford Morrison
Incision under right subcostal margin e.g. Cholecystectomy (open)
Incision in right iliac fossa e.g. Appendicectomy
Oblique incision centered over McBurneys point- usually appendicectomy (less cosmetically acceptable than Lanz
Rooftop incision
Transverse supra pubic, primarily used to access pelvic organs
Groin incision e.g. Emergency repair strangulated femoral hernia
Extraperitoneal approach to left or right lower quadrants. Gives excellent access to iliac vessels and is the approach of choice for first time renal transplantation.